Succeeding in the Biomedical Admissions Test (BMAT): A practical guide to ensure you are fully

176

Transcript of Succeeding in the Biomedical Admissions Test (BMAT): A practical guide to ensure you are fully

Page 1: Succeeding in the Biomedical Admissions Test (BMAT): A practical guide to ensure you are fully
Page 2: Succeeding in the Biomedical Admissions Test (BMAT): A practical guide to ensure you are fully

ContentsAbout the Publisher V

Reaching your Goal VI

Free Companion Material Vlll

About the Authors IX

Dedication X

Preface Xl

Chapter 1 Introduction to the BMAT 1What is the BMAT? 2Contents of the BMAT 3How is the BMAT scored? 5Who uses the BMAT? 6How do I arrange my BMAT? 7Preparing for the BMAT 9

Chapter 2 Succeeding in the BMAT 11Practice makes perfect 12What are multiple choice tests? 13Tips for succeeding in the BMAT 14

Chapter 3 Section 1: Aptitude and Skills 17Overview of Section 1 18Preparing for Section 1 19Sub-section: problem solving examples 20Sub-section: understanding argumentexamples 26Sub-section: data analysis and inferenceexamples 35

Chapter 4 Section 2: Scientific Knowledge andApplications 49Overview of Section 2 50Preparing for Section 2 50Section 2: worked examples 52

BPP@ iiiLEARNING MEDIA

Page 3: Succeeding in the Biomedical Admissions Test (BMAT): A practical guide to ensure you are fully

BPP~LEARNINGMEDIAW

iv

Contents

Chapter 5 Section 3: Writing Task 57Overview of Section 3 58Preparing for Section 3 59Tips for writing essays 60Section 3: mark scheme 62Section 3: worked example 65Section 3: practice questions 74

Chapter 6 Full BMAT mock 81Mock Section 1:Aptitude and Skills-60 minutes 82Mock Section 2: Scientific Knowledgeand Applications - 30 minutes 105Mock Section 3: Writing Task -30 minutes 119

Chapter 7 Full BMAT mock test answers andjustifications 121Mock exam paper answers: Section 1 122Mock exam paper answers: Section 2 150

Page 4: Succeeding in the Biomedical Admissions Test (BMAT): A practical guide to ensure you are fully

BPP~LEARNING MEOlA W

vi

I first began mentoring aspiring medical students seven yearsago when it was clear that many individuals were not gainingaccess to the help and support they required to successfullyapply to medical school. It was with this in mind that Iembarked on publishing our Entnj to Medicnl School Series toprovide a clear insight into the various facets of successfullygetting into medical school.Whether it is helpwith choosingthe

The decision to follow a career in Medicine is something thatshould not be taken lightly and you should undertake carefulresearch to ensure it really is for you. A career in Medicine isnot for everyone and I would urge readers to ensure they haveundertaken sufficientwork experiencetogain a balancedinsightinto what becoming a doctor really entails.

There is no greater privilege than being responsible for leadingthe treabnent of patients and sharing in their recovery.Therearefewother careers that provide such diversity on a daily basis.Apassion forhelping others, clearcommunicationskillsespeciallyempathy, excellent team working and leadership qualities aswell as the ability to strike a work-lifebalance are all skills thatan accomplished doctor should possess.

The process of applying to medical school can be a somewhatlong and arduous process but the rewards of a career withinMedicine are infinite. EPPLearningMedia and EPPUniversityCollegeSchoolofHealth are committed to supporting aspiringand current doctors to progress their career through ourcomprehensive range of books, personal development coursesand degree programmes. I often say there is no other vocationthat provides such breadth and depth of career options for theindividual to followand specialisein.Whether it is the fastpacednature of the A&Edepartment or the measured environment ofPathology, there is something for everyone.

Reaching your Goal

Page 5: Succeeding in the Biomedical Admissions Test (BMAT): A practical guide to ensure you are fully

viiBPP~J.f.ARNING MEDIA W

Matt GreenSeries Editor - Entry to Medical School

Medical Publishing Director

I would like to take this opportunity to wish you the very bestof luck with applying to medical school and hope that you passon some of the gems of wisdom that you acquire along the wayto other aspiring medics.

right medical school, how to prepare an outstanding personalstatement or how to succeed in your medical school interview,our comprehensive range of books provide the advice that isso often hard to find.

Reaching your Goal

Page 6: Succeeding in the Biomedical Admissions Test (BMAT): A practical guide to ensure you are fully

ixBPP~LEARNING MEDIA W

Matt Green, BSc (Hons), MPhilMatt Green has spent the last seven years helping thousandsof individuals prepare for and pass their BMAT.Matt has nowused this extensive experience to write this book with the aimof assistingprospectivemedical,dental and veterinary studentsto be successful in the BMATas part of their application touniversity.

Nicola Hawley, BSc (Hons)After completing her BScin Biochemistry at Leeds UniversityNicolaaccepted a researchpost within the SchoolofBiomedicalSciences at Nottingham University. This involved workingin a multidisciplinary team alongside medical professionalsresearching the therapeutic potential of G Protein CoupledReceptors,inparticularBeta-landBeta-2Adrenoreceptors.Nicolanow teaches sciencewithin a secondary school environment.

About the Authors

Page 7: Succeeding in the Biomedical Admissions Test (BMAT): A practical guide to ensure you are fully

xiBPP~LEARNING MEDIA 'U

The information in this book should be used in conjunction withinformation fromCambridge Assessment (www.admissionstests.cambridgeassessment.org.ukl adt/bmat) and any guidance givento you by the institutions you intend to apply to.

The book also contains a full mock of the BMAT, enablingyou to gain an appreciation of how you will be examined. Werecommend that this test is performed under exam conditions,in order to fully prepare yourself for the real thing. To gain thefull benefit of this book we recommend that you visit our websiteto download a free answer sheet to use when working throughthe mock test questions (www.bpp.com/freehealthresources).

This book covers all three sections of the BMAT, providingsuggestions on how to prepare for each section and workedexamples of the types of questions you will encounter duringthe test.

This book is intended to help prospective medical and veterinarystudies students prepare for their BioMedical Admissions Test(BMAT). Even though the BMAT is only a requirement fora limited number of medical and veterinary schools, it is anessential part of the assessment process for those universitiesand the results will have a direct impact on your likelihood ofacceptance.

Preface

Page 8: Succeeding in the Biomedical Admissions Test (BMAT): A practical guide to ensure you are fully

-o 0

"1/'(" •""t"

Introduction tothe BMAT

Chapter .1

Page 9: Succeeding in the Biomedical Admissions Test (BMAT): A practical guide to ensure you are fully

BPP~LEARNING MEDIA W

2

This book will explore the BMATin detail, examining the typesof questions which will be asked, suggesting ways to prepare for

While the BMATwill assess you on many of the skills requiredto be a good doctor or vet, the test will primarily assess how wellyou are likely to cope with the rigours of university. Researchinto student performance in the BMAThas shown that the testsprovide a good indication of how a student will perform atundergraduate level in Medicine and Veterinary Science. Thismeans that the universities will place a great deal of emphasison your results when deciding whether or not to offer you aplace, so it's important to prepare yourself for the exam as muchas you possibly can!

The BMAT, or BioMedical Admissions Test, was designedby Cambridge Assessment to help universities to refine theiradmissions processes. With thousands of applications to studyMedicine and Veterinary Science each year and limited places,universities needed a way to differentiate between students withsimilar levels of ability and suitability. The BMATprovides away for students from all educational backgrounds, includingmature and overseas students, to be assessed by using equalstandards.

What is the BMAT?Everyone knows that you have to have good scientificknowledgeto be a successful doctor or veterinarian, but there's more to itthan just achieving good grades in your science subjects. Youneed to be able to demonstrate excellent problem-solving skills,the ability to reason, and the ability to use all your scientificknowledge in the right way. You also need to be able to performunder pressure, while expressing your arguments and ideas ina dear and concise way that others can understand; the BMATexamines all of these attributes.

Introduction to the BMAT

Chapter 1

Page 10: Succeeding in the Biomedical Admissions Test (BMAT): A practical guide to ensure you are fully

3BPP~LEARNING MEDIA W

• Generalise or make logical deductions based on numericaland / or graphical data

• Identify, extract and understand meanings from long orcomplex texts

This problem-solving section will test your ability to:

Section 1: Aptitude and Skills (1 hour)During your degree, you will often face difficult problemswhich require you to assess a given set of information and drawconclusions based on your logic, reasoning and understandingof raw quantitative data.

Youwill have two hours to answer a total of 63 questions fromthe following sections:

Contents of the BMATThe BMAT is a paper-based assessment divided into threesections, each of which is designed to test specific knowledgeand abilities. Unlike the GCSEand A level exams you're alreadytaking, the BMATwill be testing you largely on basic skills andknowledge that you should already possess, rather than youracademic ability. The test will examine a variety of qualitiesand abilities, including your ability to read formal English andfollow written instructions, using different types of questionthroughout each section.

the exam and providing worked examples to demonstrate howto answer the questions. Wewill also provide sample questionsfor you to attempt yourself, enabling you to better understandthe BMAT and what will be expected of you on the day. Theultimate aim is that you will be as prepared as you possibly canbe for the exam and that you will be one step closer to achievingyour goal of studying at university.

Introduction to the BMAT

Page 11: Succeeding in the Biomedical Admissions Test (BMAT): A practical guide to ensure you are fully

BPP~LEARNING MEOlA '"

4

Communicate your knowledge, arguments and deductionsthrough clear and concise language or diagramsCreate rational, logical and well-rounded arguments basedon medical or scientific concepts

This section will test your ability to:

Section 3: Writing Task (30 minutes)As a student, and subsequently as a doctor or vet, you willencounter many statements, assumptions and concepts; youwill have to be prepared to challenge these concepts, examiningthem from every angle to develop an understanding of not onlyhow but why things are as they are.

• Approach problems logically, using evidence-basedreasoning

• Recall basic theoretical knowledge under pressure• Apply your knowledge of a variety of disciplines, many of

which you may not have had a great deal of exposure to.

This section will test your ability to:

Section 2: Scientific Knowledge and Application(30 minutes)The second section, which is also multiple choice, will test yourability to apply scientific knowledge to given scenarios. Thequestions will be drawn from GCSE level Science (up to higher­level double award) and Maths.

It is important to note that each question in this section maytest you on two or more of the above.

• Read and understand simple quantitative data, innumericalor graphical form, and produce simple graphs and diagramsbased on this data

Chapter1

Page 12: Succeeding in the Biomedical Admissions Test (BMAT): A practical guide to ensure you are fully

5BPP~LEARNING MEDIA W

Every essay is double marked. If the marks for content are thesame, or at a neighbouring point on the scale, an average of

Your essay in Section 3 will be given a score on a scale betweeno and 5. The score will reflect the quality of content of the essay,and will be combined with a score for quality of written English:A, Cor E.

How is the BMAT scored?You will record your answers on a separate sheet that will bemarked electronically. For each individual question asked inSections 1and 2 a total of 1mark is available. Your marks for eachsection will be totalled and converted to the BMAT scale. Thiswill give you a score between 1 {low) and 9 (high). Your scorewill be given to one decimal place. The majority of candidateswill score approximately 5 on each section which equates to ascore of approximately 50%. Stronger candidates will score 6,and only exceptional candidates 7 and above.

Examples of all three answer sheets can be found on the BMATwebsite at:www.admissionstests.cambridgeassessment.org.uk/ adt/bmat/Test+Preparation

When sitting the BMAT you will be provided with an answersheet. For Sections 1 and 2 this will either involve circling thecorrect multiple choice answer(s), or writing short verbal ornumerical answers directly into spaces provided. The Section 3answer sheet is discussed later on in this book.

Later in this book we will break each section down, givingexamples of how to answer each section effectively and whatyou need to do to prepare yourself fully.

• Adopt a critical approach and consider alternative argumentswhen formulating a point of view.

Introduction to the BMAT

Page 13: Succeeding in the Biomedical Admissions Test (BMAT): A practical guide to ensure you are fully

BPP~LEARNING MEDIA W

6

Currently, the BMAT is a requirement for entry on to thefollowing courses:

Ultimately, each university will use the results in conjunctionwith your UCAS application, personal statement and interviewto decide whether or not to offer you a place on its Medicine orVeterinary Science course. The more prepared you are for theBMAT,the better chance you have of winning that universityplace!

Each university uses the results of the BMATin their own way,and will accept different scores as part of their admissionsrequirements - you should look at your individual courserequirements to see what score your university will require.

• University of Cambridge (www.cam.ac.uk/)• University of Oxford (www.ox.ac.uk/)• Imperial College London (www3.imperial.ac.uk/)• University College London (www.uc1.ac.uk/)• Royal Veterinary College (www.rvc.ac.uk/)

Who uses the BMAT?Currently, the BMAT is used by the following universities aspart of their admissions process:

Your results will be released in the second half of November.The score you are awarded is final and cannot be challenged.

the two is awarded. If the marks for writing quality are thesame, or at a neighbouring point on the scale, the scores arecombined: AA = A, AC =B, CC = C, CE =D and EE = E. If themarks differ significantly the essay will be marked for a thirdtime and a final mark given is checked by a senior member ofthe Cambridge Assessment staff.

Chapter 1

Page 14: Succeeding in the Biomedical Admissions Test (BMAT): A practical guide to ensure you are fully

7BPP~LEARNING MEDIA W

How do I arrange my BMAT?Even though you are required to take the BMAT as part of youruniversity application, it is ultimately your responsibility toarrange the test by the required date.

Royal Veterinary College (R84)• Veterinary Medicine (0100)• Combined Degree Programme (0101)• Graduate Accelerated Veterinary Medicine Programme

(0102)

University College London (U80)• Medicine (A100)

Imperial College London (ISO)• Medicine (A100)• Biomedical Sciences - three-year course (B900)• Biomedical Sciences with Management - four-year course

(B9N2)

University of Oxford (033)• Medicine (A100)• Biomedical Sciences (BC98)

University of Cambridge (COS)• Medicine (A100)• Graduate Course in Medicine (A101) - note that the BMAT

is not essential for this course• Veterinary Medicine (0100)

Introduction to the BMAT

Page 15: Succeeding in the Biomedical Admissions Test (BMAT): A practical guide to ensure you are fully

BPP~LEARNING MEDIA W8

CostsTaking the BMATdoes incur a small fee; the fee will depend onyour status and when you apply, and you should also ensurethat the centre you are registered with does not charge its ownadministrative fees on top of this.

Mid/end November Results are released to candidates viat~~_~!sults On_!inesystem

Early November

Mid October

Early September Centres start making candidateentriesStandard closing date (standardentry fees apply until Spm BST)Last date for late entries (late entryfees apply untilSpm BST)The date of the actual BMAT exam

Early October

..,

Exam datesJust like your UCAS application, your registration for the BMATmust be completed by a set deadline. The important dates forentry in 2012 are:

www.admissionstests.cambridgeassessment.org.uk/ ad t / bmat /registration

Registering for the examThe first step is to register with a BMATcentre, as it is not possibleto register yourself to take the test. Ifyou are attending school orcollege, talk to your Examinations Officer about the possibilityof having the test arranged on your behalf. If your school orcollege is unable to arrange the test or if you are not currentlyattending school or college, then you will need to register withan open testing centre. You can find a list of centres, togetherwill full contact details, at:

Chapter 1

Page 16: Succeeding in the Biomedical Admissions Test (BMAT): A practical guide to ensure you are fully

9BPP~LEARNING MEDIA W

There are techniques which will help you in each particularsection, but there are also more general tips which will help youto prepare for the exam as a whole. These include:

Although the BMATis specifically designed to negate the needfor extra revision, this doesn't mean that there aren't ways toprepare for each section of the exam. Later on in this book wewill explore each section in further detail and suggest ways inwhich you can better prepare yourself for the types of questionyou will be answering at each stage.

Preparing for the BMATAs an aptitude test, the BMATwill not be examining your abilityto understand and remember complex scientificinformation. Theuniversities will be able to determine your level of knowledgefrom your A level results; instead, they will be looking to assessyour innate intelligence and personal ability coupled with yourscientific understanding at GCSE level. This means that youwon't necessarily need to learn a whole lot of new informationfor the exam, but it also means that you can't just cram for aweek and hope to get a high mark!

Students who are eligible for the full Maintenance Grant, fullAdult Learning Grant, Income-based Job Seeker's Allowance orIncome Support may be eligible to apply for reimbursement ofthese fees. EU candidates who receive equivalent funding mayalso be eligible for reimbursement.

£42.50 Standard entry fee for a UK / EU candidate£72.50 Late entry fee for a UK / EU candidate£72.50 Standard entry fee for non-UK / EU candidate£102.50 Late en!ry.fe~ for n~~UK / E~andidate

Costs for 2011BMATentry were:

Introduction to the BMAT

Page 17: Succeeding in the Biomedical Admissions Test (BMAT): A practical guide to ensure you are fully

10

Always bear in mind that the BMAT is not designed to trickyou or to catch you out in any way. You are being tested onabilittj rather than knowledge. Although some sections do testyour knowledge, you will only be questioned on topics youshould already know quite welL As such there is no 'right' or'wrong' way to approach this exam, though the hints and tipswe provide throughout this book should enable you to be asprepared as possible for the exam.

• Reading further books on critical thinking and problem­solving (see the reading list at the end of this book)

• Working through old exam papers to improve your generalknowledge on scientific and mathematical problems

• Keeping an eye on the news and on journals to learn aboutnew developments or current debates in medicine

• Completing mock BMATpapers and practice questions tofamiliarise yourself with the exam

Chapter 1

Page 18: Succeeding in the Biomedical Admissions Test (BMAT): A practical guide to ensure you are fully

0 0"'If! 0 0''If! a"'f! ~- <....

.::.

Succeedlng in'the BM'AT

Chapter 2

Page 19: Succeeding in the Biomedical Admissions Test (BMAT): A practical guide to ensure you are fully

BPP~LEARNING MEDIA W

12

Itmay therefore be valuable to timeyourself when you undertakethe mock test at the end of this book. This way you will be ableto enhance your time management skills, togetherwith increasingyour confidence and alleviating any anxiety you may have. Theaim of this book is to ensure that, on the day of your test, youare faced with something you are already familiar with.

Each of the sections is individually timed, therefore it is notpossible for you to make up for lost time in the other remainingsections. It is vital that you complete each section fully asyou progress through the test and do not leave any questionsunanswered. Unlike conventional tests you are unable to go backand finish off unanswered questions from previous sections.

However, this is not to say that the BMAT test is of an easynature, otherwise it would not be a valid measure in the selectionprocess. Although the BMATmay measure general ability youwill find that you only have a limited amount of time for eachsection.

The following chapters will enable you to practise each of thedifferent sections which together form the BMAT.Each chaptercontains questions to provide you with an insight into whatyou will be faced with in the test, and aims to help you to putinto practice what you Jearn. This will enable you to familiariseyourself with the format and style of the BMATand hopefullyhelp you to realise that most of the questions in the test will beof general ability.

Practice makes perfectAs with any test it is essential to practise example questions toensure you are familiar with the structure and type of contentyou will be tested on. The BMATis no exception despite whatpeople may tell you!

Succeeding in the BMAT

Chapter 2

Page 20: Succeeding in the Biomedical Admissions Test (BMAT): A practical guide to ensure you are fully

13BPP~LEARNING MEDIA W

forLead-in questionThis is the question which identifies the exact answer,example:

'There are 100 students who go on a school trip to a sciencepark.'

StemThisis generallyan introductory statement, question or passageof relevant information which elicits the correct answer. Thestem on the whole provides all the information for the questionor questions which will follow,for example:

The sectionswithin the BMATare based on an answer formatknown as 'A type questions', which is the most commonly useddesign in multiple choice tests. This specific design helps tomake transparent the number of choices which need to beselected.These questions usually consist of a 'Stem and lead-inquestion' which is followedby a 'series' of 'choices', or a requestfor a short numerical or verbal answer.Toillustrate this, belowis an example of a question you may be faced with:

What are multiple choice tests?TheBMATis set out in amultiple choiceor short answer format,with the exceptionofSection3whichwewill address later in thisbook. These types of tests are commonly used within the fieldof selection and assessment.The test questions are designed totest a candidate's awareness and understanding of a particularsubject.

Thisbook culminates with a full mock test for you to completeunder timed conditions.An answer sheet can be downloadedfor free from www.bpp.com/freehealthresources to help youmake the most of this book.

Succeeding in the BMAT

Page 21: Succeeding in the Biomedical Admissions Test (BMAT): A practical guide to ensure you are fully

BPP~LEARNING MEDIA W14

•• ,. , .'. ~w ~ ••••••_,..,.,....•••••_.¥..~.....~ .,.,." .. ,.. ... . . . ......

To help you with your time management ensure youtake a watch or timer with you to the test.Read, and re-read the question to ensure you fullyunderstand what is being asked, not what you want tobe asked.Eliminate any incorrect answers you know areincorrect.Read the question and try to answer it before lookingat the choices available to you.Ensure you record your answers clearly as Sections 1and 2 are marked by computers.

Tips for succeeding in the BMAT

Alternatively you may simply be asked to state the number inthe space provided.

D 65

A 25B 67C 35 - Correct answer (35% of 100 students = 35 female

students)

ChoicesIna multiple choice test, the choices will generally consist of onecorrect answer. However, depending on the type of question,you may be required to find two or even three correct answers.Wherever there are correct answers there are also incorrectanswers, which are also known as the' distracters'. For the aboveexample, typical choices could be as follows:

'35% of tile students were female, how IIlfllly female studentswere there?'

Chapter 2

Page 22: Succeeding in the Biomedical Admissions Test (BMAT): A practical guide to ensure you are fully

15BPP telLEARNING MEDIA W

• When you enter a written answer (particularly inSection 3) ensure you write clearly and legibly.

• Ensure when changing your answers you erase the oldanswer thoroughly.

• Do not spend too much time on one question -remember you only have a set amount of time persection so, as a rule of thumb, you should spend xamount per question (x = time of section + number ofquestions).

• Do not keep Changing your mind - research hasshown that the first answer that appeals to you is oftenthe correct one.

• If you cannot decide between two answers - lookcarefully and decide whether for one of the optionsyou are making an unnecessary assumption - trustyour gut instinct.

• Always select an answer for a given question evenif you do not know the answer - never leave anyanswers blank.

• Pace yourself - you will need to work through the testat the right speed. Too fast and your accuracy maysuffer, too slow and you may run out of time. Use thisbook to practise your time keeping and approach toanswering each question - you need to do what worksfor you, not what might work for someone else.

• Try to leave some time at the end of each section tocheck your answers.

• To familiarise yourself with the way the test will beconducted visit the BMATwebsite which also containsfurther practice questions.

• Remember you will only be awarded marks for correctanswers, and marks will not be deducted for incorrectanswers. Therefore answer every single question, evenones you are unsure of.

Succeeding in the BMAT

Page 23: Succeeding in the Biomedical Admissions Test (BMAT): A practical guide to ensure you are fully

BPP~LEARNING MEDIA W

16

- .When you take the test, listen carefully to theadministrator' 5 instructions.If you are unsure about anything, remember to ask the e

test administrator before the test begins. Once the clock :begins ticking, interruptions will not be allowed.Youmay be presented with a question which yousimply cannot answer due to difficulty or if thewording is too vague. If you have only approximately90 seconds per question, and you find yourself .spending five minutes determining the answer for each :question then your time management skills are poorand you are wasting valuable time.Do not be alarmed if the test is different from thepractice papers you have worked through, rememberyou are being tested primarily on aptitude.

Chapter 2

Page 24: Succeeding in the Biomedical Admissions Test (BMAT): A practical guide to ensure you are fully

c» o"'f1ff •~

• 0(i..i U

Section 1:Aptitude andSkills

Chapter 3

Page 25: Succeeding in the Biomedical Admissions Test (BMAT): A practical guide to ensure you are fully

BPP~LEARNING MEDIA W

18

Each sub-section will test you using different types of questions(these are covered in the individual sections of the chapter). Thecommon theme of the questions is that you will be presentedwith a set of data - numerical, textual or graphical - and will

• Problem solving (30 minutes)• Understanding argument (15 minutes)• Data analysis and inference (15 minutes)

Section 1 is divided into three further sub-sections:

A maximum of 35 marks are available in this section.

Section 1 will test your ability to analyse and reason throughvarious mathematical, logical and observational scenarios. Youwill have a total of one hour to complete 35 multiple choicequestions. You will be provided with an answer sheet, where youwill circle the correct multiple choice answer(s), or write shortverbal or numerical answers directly into the spaces provided.One very important thing to remember is that you are not allowedto use a calculator for this section, so all working out must bedone on the exam paper itself or in your head.

Overview of Section 1Problem-solving skills are essential for any medical or veterinarystudent; you will often be called upon to resolve problemsarising from complex ideas or concepts, most likely underpressure, and will have to call on your powers of logical andanalytical thinking to succeed. The practice of evidence-basedmedicine is a current issue of debate in the medical professionand being able to make judgements based on limited data is adifficult technique to perfect. As a doctor, this skill will be vitalin making on-the-spot decisions, many of which could be life­or-death, based on the evidence presented to you.

Section 1: Aptitude and Skills

Chapter 3

Page 26: Succeeding in the Biomedical Admissions Test (BMAT): A practical guide to ensure you are fully

19BPP~LEARNING MEDIA W

Section 1 focuses on technique, not knowledge; the best way tosucceed is to focus on how to answer the questions. The mostobvious way to do this - practice! Spend as much time as youcan working through example questions and papers so that youknow exactly what will be asked of you in the exam. Whenever

• Address the type of complex problems you could be facedwith in your future career

• Interpret scientific data and concepts• Draw conclusionsfromarguments and information presented

to you

This section will test you on your ability to:

Preparing for Section 1As this section involves purely 'evidence-based' questions,there are no subjects to revise beforehand. While this may makepreparation sound more difficult, there are ways in which youcan prepare for Section 1.

Thenext few sectionswill explorehow you canprepare for SectionI, and will work through some examples to allow you to get togrips with the kinds of questions you will be answering.

Note that the sections will not appear in the exact order above,but will appear randomly from each sub-section. The timings areto give an idea of what proportion of Section 1will be taken upby each sub-section, and thus roughly how long you should betaking on each type of question. It is also important to note thata question may assess you on two or more of the above skills.

be asked to come to a conclusion based entirely on the evidencepresented to you. If a question is multiple choice, you will haveto narrow down your answer choices to find the answer (oranswers if the question asks for more than one) which best fitsthe question. Eliminating incorrect answers first can help youto identify the correct answer.

Section 1: Aptitude and Skills

Page 27: Succeeding in the Biomedical Admissions Test (BMAT): A practical guide to ensure you are fully

BPP~LEARNING MEDIA W

20

In the following pages we will examine the types of questionsyou will encounter in the problem solving sub-section.

• Identifying the appropriate (and inappropriate) data in theinformation provided to enable you to answer the question- an important skill required of medical professionals

• Recognising patterns and trends within data and reapplyingthis information to new data sets

• Selecting the correct approach or formula to determinethe correct answer from the information provided, oftencombing multiple data sets - always read the question inits entirety to ensure that you identify the bigger picture

Sub-section: problem solvingThis sub-section will include both numerical and logical problems(equivalent of Key Stage 4 curriculum), designed to assess yourability to think a situation through logically and precisely.Youwillneed to look at the information provided and work systematicallytowards the answer by first establishing all of the possibilities,then eliminating them one by one until you reach your answer.Some of the skills required for this sub-section are:

Youcould also spend some time reading up on problem-solvingor analytical techniques. This isn't essential, and won't providethe same benefits as practising the questions directly, but it mightgive you ideas to help spot the correct answers or to improveyour timing. Be careful though, as many of these books willprobably be more in-depth than you need, and may exploreskills that aren't completely relevant to the BMAT.

possible, practise under timed conditions; there's no point inknowing how to answer the questions if you can't do it in theallotted time, so find a quiet space, set your stopwatch and seehow well you can do. Each time you practise, you should getbetter at answering the questions within the required time.

Chapter 3

Page 28: Succeeding in the Biomedical Admissions Test (BMAT): A practical guide to ensure you are fully

21

Now it's just a process of elimination. At no point could Chrisbe ahead of Edward, so that rules out Option C. He also cannotbe ahead of Brian, which rules out Option D. He is definitelybehind Brian, but even though he is likely to be behind Darren,this isn't definite which rules out Option A. The most likelyanswer, then, is Option B.

Brian, Andy, Edward, Darren, Chris

Brian, Andy, Darren, Edward, Chris

Brian, Andy, Edward, Chris, Darren

AnswerThese questions are fairly self-explanatory, but don't be caughtout by trying to work it out in your head. The best way towork through this is to draw it all out and work through itvisually. This is where the blank pages in the exam paper comein useful! Based on what we're given above, there are a numberof constants: no one is ahead of Brian; Andy is behind Brian;and Chris, Darren and Edward are behind Andy. This leads tothree possibilities:

A Behind Brian and ahead of DarrenB Behind Andy, but not necessarily behind DarrenC Behind Darren and ahead of EdwardD Ahead of Brian, but not necessarily behind Andy

Chris must be:

Example question 1Five friends are queuing up together for tickets to their favouriteband. Darren is behind Andy, who is behind Brian. Edward isin front of Chris, but behind Andy.

Section 1: Aptitude and Skills

Page 29: Succeeding in the Biomedical Admissions Test (BMAT): A practical guide to ensure you are fully

BPP~LEARNING MEDIA W

22

The passage is formed of two separate sentences. The first isa definite, signified by 'will'. The second is more conditional,signified by 'may'. This is the first step to answering this question- differentiating between what 'will' and what' could' happen.

If someone suffers a migraine, they will experience a painful headache.Most, though not all, sufferere ofmigraines may also sufferfrom nausea,flashing lights, sickness, and sensitivity to light and sound.

AnswerLet's examine this question in a little more detail. First, thequestion itself:

A 3,2, 1B 2,3, 1C 1,2,3D 2, I, 3E 3, I, 2F I, 3, 2

1. Anyone suffering from a migraine will experience nauseaand sensitivity to light and sound

2. Anyone experiencing a migraine will experience a headacheor sickness

3. Anyone suffering from amigraine will experience a headache,possibly accompanied by flashing lights and sickness

Based on the information above, which option, from A to F,correctly places the following sentences in order of probability,with the most probable first?

Example question 2If someone suffers a migraine, they will experience a painfulheadache. Most, though not all, sufferers of migraines may alsosuffer from nausea, flashing lights, sickness, and sensitivity tolight and sound.

Chapter 3

Page 30: Succeeding in the Biomedical Admissions Test (BMAT): A practical guide to ensure you are fully

23BPP~lEARNING MEDIA W

2. Anyone experiencing a migraine will experience a headache orsickness

The first point of note is that this is a definite statement ('will').Looking back at the question, we are told that 'most' migrainesufferers will experience nausea and sensitivity to light. The onlydefinite symptom of a migraine is a headache, so this optiondoesn't seem entirely probable.

1. Anyone suffering from a migraine will experience nausea andsensitivity to light and sound

Let's look at the three sentences and determine the probabilitiesof each one:

In this case, you are being asked to look at options A to F anddetermine which one best describes the order of the threefollowing sentences. Pay particular attention to that last part ofthe question; the option you are looking for should place themost probable first. This will not always be the case, and somequestions may ask you to place the least probable first. Makesure you read the question fully and understand what's beingasked of you.

Which option, from A to F, correctly places the following sentence in,order of probability, with the most probablefirst?

The second part of the question is telling you what you arerequired to do:

It's vital to remember that your answer should be based on theevidence presented in the question, not on what you may alreadyknow about the subject. You may be a migraine sufferer and youmay always experience a headache and nausea together, but thatinformation isn't relevant to the question. Only use what's rightin front of you, even if you think it is factually incorrect.

Section 1: Aptitude and Skills

Page 31: Succeeding in the Biomedical Admissions Test (BMAT): A practical guide to ensure you are fully

24

(i) His call out charge(ii) His hourly rate

Calculate:

Example question 3A plumber charges a flat rate call out charge and an hourly ratefor every job he does. In one day he does two jobs, one for fourhours which earns him £73, and one for five and a half hourswhich earns him £91.

Based on the above arguments, the answer is option A: 3, 2, 1as the most likely order of statements.

Again, we have a definite element ('will'), but we also have aconditional element ('possibly accompanied' acts as a 'may').The first part we can deduce to be correct, as we can deducethat a headache is a definite symptom of a migraine. However,we can also deduce that flashing lights and sickness are alsopossible accompanying symptoms. This is where we mustdecide on where this statement fits in terms of probabilities; theprobability of having a headache and accompanying symptomsis higher than that of having a headache or other symptoms, sostatement 3 is more probable than statement 2.

3. Anyone suffering from a migraine will experience a headache,possibly accompanied by flashing ligh.ts and sickness

Like the first statement, this statement has a definite element('will'). However, there is also a conditional element ('or'). Considerthe alternatives of the 'or' element; according to the informationprovided, a migraine could consist of a headache alone, but itcould not consist of sickness alone. As such, this statement iseither highly probable or highly improbable. However, becausethe statement includes an 'or' and not an 'and', this statementstill carries a higher probability than statement 1.

Chapter 3

Page 32: Succeeding in the Biomedical Admissions Test (BMAT): A practical guide to ensure you are fully

25BPP~LEARNING MEDIA 'U

So his call out charge is £25, and his hourly rate £12.

4(12) + y = 73

48 + Y = 73

Y = 73 - 48y= £25

We now substitute the value of x into Equation 1:

5.5x + Y = 91

5.5x + (73 - 4x) = 91

1.5x + 73 = 91

1.5x = 18

x = £12

We can now substitute y into Equation 2.

4x+y=73

Y = 73 -4x

If we rearrange Equation 1 we can make y the subject:

And his second: 5.5x + y = 91 (Equation 2)

So for his first job: 4x + y = 73 (Equation 1)

AnswerThe first thing to do is to express his earnings as an equation,where x can be his hourly rate and y his flat call out charge.

Section 1: Aptitude and Skills

Page 33: Succeeding in the Biomedical Admissions Test (BMAT): A practical guide to ensure you are fully

BPpALEARNING MEDIA W

26

What is the main conclusion of the informationprovided?Which of the below options indicate a flaw in the passageabove?Which of the below options would further strengthen theabove conclusion?What is the main assumption of the argument presented inthe passage?

Questions may ask for the following information:

• Inference - determining reliable concl usions from theinformation provided and, where applicable, identifyingunsafe ones.

• Ability to analyse - determining the motivations, themes,assumptions and any conclusions in a given set ofinformation.

• Effectively evaluating information - determining flaws anderrors in given information, identifying pros and cons inan argument and whether the information supports thesearguments.

The main skills this sub-section will evaluate you on are:

Sub-section: understanding argumentThis sub-section will use short passages of text, usually justa paragraph, or small data sets, to test your ability to quicklyidentify and understand arguments. There are many ways inwhich these questions can be phrased, but the core principlebehind each question will be the same: assess a given set ofinformation and, using your skills of reasoning, work towardsthe answer you are asked to find. This may be multiple choiceor may request a short written or numerical answer. It is vitalyou only use the information you are given, and not your priorknowledge.

Chapter3

Page 34: Succeeding in the Biomedical Admissions Test (BMAT): A practical guide to ensure you are fully

27BPP~LEARNING MEDIA W

Succeeding in the understanding argument sub­section• Ensure the answer you give is determined solely on the

information containedwithin the passage.• Look out for misleading words such as 'all', 'everything'

and 'completely' - these are specific types of words whichsuggest that the whole of a particular object,person, areaor group are wholly affected.

• Other misleading words include 'virtually', 'almost','particularly', 'nearly' and' closeto' - thesearewords which

• Conclusion:a proposition concluded or inferred from thepremises of an argument

• Assumption: something taken for granted• Explanation:something that explains; a statement made to

clarify something and make it understandable• Inference:derived by reasoning;conclusionsor judgements

made / based on premises or evidence• Implication:the relationship between two propositions, or

classesof propositions, by virtue of which one is logicallydeducible from the other

• Flaw:indicationthat a givenmeaning resultsin an argumentor conclusionbeing unjust

• Justification: a reason, fact, circumstance, or explanationthat justifies or defends

• Reason:a statementpresented in justificationor explanationof a belief or action

• Ambiguous: of doubtful or uncertain nature; difficult tocomprehend, distinguish, or classify

• Anomaly:an odd, peculiar, or strange condition, situationor quality

• Discrepancy:an instance of differenceor inconsistency• Definition:the act of defining or making definite, distinct,

or clear

It is important to have a clear understanding of the definitionof some of the terms used in these question types:

Section 1: Aptitude and Skills

Page 35: Succeeding in the Biomedical Admissions Test (BMAT): A practical guide to ensure you are fully

BPPteILEARNINGMEDIA W

28

••

refer to something close to happening rather than actuallyhappening.It is very important that you read the passages very carefully.One common mistake that candidates often make is toallow their previous knowledge on a subject to interferewith and bias the information and facts that are presentedin the passages (often these are of a conflicting nature).Remember that each of the passages is deliberatelymanipulated to influence the candidate to a particularperspective or point of view.Often you may find that a passage states information whichmay subsequently alter or be contradicted further on in thepassage. Ensure that you note any changes or contradictionsand reflect these when selecting your answers.Do not waste too much time thinking about a difficultquestion. All questions are marked equally, therefore adifficult question will not be worth more than an easyquestion and vice versa. If you are having difficultyunderstanding a passage, flag it and move on to the nextpassage and come back to it later.Remember time management is key throughout the test.If you find a question particularly difficult you can flag itso that you can return to it before you move onto the nextsubtest.Attempt all questions as you will not be penalised for gettingquestions wrong, but you will lose marks if you leave anyanswers blank.Learn to manage your time efficiently. Go through practicemock papers and time yourself as if you were in a real exam.By familiarising yourself with the types of questions youwill be faced with you will be able to analyse where yourweaknesses are and improve.Read through newspapers and various other sources ofliterature which use elaborate and detailed language. Thiswill enhance your skills in reading and also enable you toconsider in-depth critical arguments and perspectives.

Chapter 3

Page 36: Succeeding in the Biomedical Admissions Test (BMAT): A practical guide to ensure you are fully

29BPP~LEARNINGMEDIA W

Which one of the following statements best summarises the aboveinformation?

AnswerThese types of question often appear to be much more difficultthan they are. As you only have a short time to complete thequestions in, the idea of reading passages of text may be daunting,but don't forget that the questions are designed to be relativelystraightforward. Onceyou've looked at the information presentedto you, let's look at what we're being asked to do:

A It is not possible to predict Type 2 diabetes in peopleB A healthy lifestyle will help prevent you from developing

diabetesC Diabetes can be caused by eating lots of sugary foodD Young, slim people are unlikely to suffer from diabetesE Obesity is a significant, though not the only, factor affecting

diabetes

Which one of the following statements best summarises theabove information?

Example question 1Diabetes canbe caused by a number of factors.The most commonform of diabetes, Type 2, is frequently linked with obesity, andexercise and healthy eating are often prescribed as a means ofmanagement. Ageing can also be a factor, and a family historyof diabetes increases the risk of developing the condition. Thecombination of causing factors can make it extremely difficultto predict the,emergence and severity of Type 2 diabetes.

The following example questions cover some of the types ofquestions you will be asked, though this is by no means anexhaustive list.

Section 1: Aptitude and Skills

Page 37: Succeeding in the Biomedical Admissions Test (BMAT): A practical guide to ensure you are fully

BPP ,e.LEARNING MEDIA \Q

30

This certainly fits the information given to us; we are told thatobesity and age are factors in diabetes, but does this summarisethe passage as a whole?

D Young, slim people are un.likely to suffer from diabetes

Obesity is put forward as a potential cause of diabetes; you mayknow that eating lots of sugary food can cause obesity, but thatisn't mentioned in the passage. Based on the information givento us, this isn't a good summary.

C Diabetes can be caused by eating lots of sugary food

This is certainly suggested, as it is stated that 'exercise andhealthy eating are often prescribed as a means of management'.However, this is only a passing mention.

B A healthy lifestyle will help prevent you from developingdiabetes

The last line of the passage states that it is 'extremely difficult topredict the emergence and severity ofType2 diabetes', rather thanimpossible. This statement is, therefore, not a good summary.

r

A It is not possible to predict Type 2 diabetes ill people

The best approach to these types of question is to quickly'l:ietermine and eliminate which of the answers is definitelyincorrect. Work your way down to the most likely answers andthen compare them to see which one looks the most accurate.Let's look through each of the statements and see which onesbest fit the information we've been given:

Make sure you read this carefully; this question is asking youto pick the one statement that best summarises the passage. Youmay be asked to pick more than one statement, so make sureyou do what the question asks.

Chapter 3

Page 38: Succeeding in the Biomedical Admissions Test (BMAT): A practical guide to ensure you are fully

31BPP~LEARNING MEDIA W

These questions demonstrate why it's important to read andunderstand what's beingaskedofyou.Unlikethe 'summarisingthe passage' question, you are being asked to highlight theassumption that is being made at the heart of the passage. Theanswer won't necessarilybe based on anything factual in thepassage,but will be something that is inferred throughout andthat ultimately affectswhether the information in the passagemakes sense.

A Only younger criminals receiveASBOsB Antisocialbehaviour is disruptiveC ASBOswill reduce levels of antisocialbehaviourD Antisocialbehaviour is becomingmore frequentE ASBOsare ineffectiveat stopping antisocialbehaviour

What is the underlying assumption of this passage?

Example question 2As a response to rising levels of antisocial behaviour, thegovernmentintroducedtheAntisocialBehaviourOrderCASBO)in1998.ASBOstarget sociallydisruptive behaviour,and areissuedfor a variety of offencesincluding vandalism and harassment.However, ASBOshave received criticism for being too 'open­ended' and for becoming'badges of honour' formany youngercriminals.

Based on the information given to us, we can deduce that themost likely answer is E.

Both the first and the last sentences of the paragraph refer tothe variety of causes of diabetes. Weare told that obesity is afrequently suspected culprit, though ageing and genetics alsoplaya part, sothis statementwould seemtobe the best summaryof the passage.

E Obesity is a significant, though not the only, factor affectingdiabetes

Section 1: Aptitude and Skills

Page 39: Succeeding in the Biomedical Admissions Test (BMAT): A practical guide to ensure you are fully

BPP te'ILEARNING MEOlA W

32

This is basically a re-wording of the first sentence; as such, it'snot really an assumption and can be quickly disregarded.

D Antisocial behaviour is becoming morefrequent

Again, the first sentence tells us that ASBOs were introducedbecause of rising levels of antisocial behaviour. We are alsotold that ASBOs are targeted towards disruptive behaviour.We are never told specifically that ASBOs will affect levels ofantisocial behaviour, but as we are told that they are a responseto increasing antisocial behaviour, we must assume that theyare working (or why else would we use them?). This seems tobe a very likely answer, but remember to look through the restof the options before deciding.

C ASBOs will reduce levels of antisocial behaviour

We're told in the first sentence that ASBOswere introduced as aresponse to levels of antisocial behaviour. In the second sentencewe are told that ASBOs target' disruptive behaviour'. From thiswe can infer that antisocial behaviour is disruptive, and whilethis is in itself an assumption, it is not the main assumptionunderlying the passage.

B Antisocial behaviour is disruptive

It is mentioned in the passage that younger criminals often viewASBOs as 'badges of honour', but there does not appear to beany kind of indication that younger criminals are specificallytargeted for ASBOs.As such, this isn't all underlying assumptionof the text.

A Only younger criminals receive ASBOs

As with other similar questions, read the passage carefully andexamine each potential assumption at a time. Eliminate theobviously incorrect answers, and then work through what isleft to make a decision.

Chapter 3

Page 40: Succeeding in the Biomedical Admissions Test (BMAT): A practical guide to ensure you are fully

33BPP~LEARNINGMEDIA W

A In the 11th month of the year CPI annual inflation was 2.1%and the price of a litre of petrol was 100 pence

B There was an overall fall of 0.2% compared to last year'sfuel prices

C Gas and electricity prices rose by 10.8%in NovemberD The main reason CPI annual inflation rose was due to

increase in the price of fuel

Which of the following statements are flawed based on theabove passage:

Gas and electricity bills remained constant with an average billescalating up to almost £600 a quarter for both types of bills.The prices of the bills remained constant up until Novemberwhen prices increased by 10.8%.

Example question 3Consumer Price Index (CPI) annual inflation - the Government'starget measure - was 2.1% in November and in the previousmonth. The main increase came from the changes in the price offuel. The average price of petrol increased by 3.5 pence per litrein November, to a standstill at £1.00per litre. When compared tolast year's prices there was an overall fall from last Novemberof 0.2 pence per litre.

Having looked through the possibilities, the most likely answeris C.

Although we can assume that ASBOs will have an effect onantisocial behaviour, as that is what they are designed to do,there is no insinuation that they are not effective. The finalsentence does detail criticisms of ASBOs, but does not suggestor insinuate that they are ineffective in any way.

E ASBOs are ineffective at stopping antisocial behaviour

Section 1: Aptitude and Skills

Page 41: Succeeding in the Biomedical Admissions Test (BMAT): A practical guide to ensure you are fully

BPP~LEARNING MEDIA W

34

= 6.7% - 3.2%

=3.5%

Inflation rate in December - inflation rate in October

AnswerA simple subtraction is required.

A 3.5%B 3.0%C 3.2%D 5.5%

What is the difference in HPI inflation between October andDecember?

Example question 4House Price Index (HPI) inflation rose to 6.7% in December, upfrom around 3.2% in October. The major causes which influenceHPI were comparable to those affecting the CPI. However, therewere additional major contributions from food and motor vehiclepurchase costs and a moderately descending role from vehicleinsurance.

Option B is therefore the correct answer.

Option D is correctly indicated in the first paragraph.

Option C is correctly summed up in the second paragraph.

AnswerOption A is correctly summed up by the first paragraph.

Option B is incorrect as the fall in fuel prices is actually 0.2pence not 0.2%.

Chapter 3

Page 42: Succeeding in the Biomedical Admissions Test (BMAT): A practical guide to ensure you are fully

35BPP~LEARNING MEDIA W

The table below shows the average house prices for regions inthe UKfrom 2005until 2008.

Example question 1Questions I, 2 and 3 refer to the data in the table below.

• Determine conclusionsfrom the information provided• Manipulate further the data presented to you to deduce

answers• Interpret complex data to determine reasons for patterns

or flaws

Foreachsetofdata and informationpresentedapproximatelyfourquestionswillbe asked relating to that data. Togetherwith someof the skillsrequired for the problemsolvingand understandingargument sub-sectionsyou will also be required to:

Sub-section: data analysis and inferenceThis sub-sectionwill test your ability to analyse statistical dataand selectwhich statementsbest describewhat the data is tellingyou. The questions will take the form of graphs and tables orverbal information, where you will need to first understandthe data presented to you before choosing the most accurateinterpretation.

Therefore the correct answer is A: 3.5%

Section 1: Aptitude and Skills

Page 43: Succeeding in the Biomedical Admissions Test (BMAT): A practical guide to ensure you are fully

BPP telLEARNING MEDIA W

36

For each we work out the difference in price between 2006 and2007, which we do simply by subtracting 2006 values fromthose of 2007.

AnswerInstead of wasting time calculating the price increases for all theregions, you need only calculate them for the options you aregiven above, as one of these must be the right answer.

A LondonB South EastC WalesD EastE South West

Which region saw the greatest percentage increase in pricebetween the years 2006 and 20077

Average price (£)

Region t008 2007 2006 2.005

North East 133,500 131,000 125,500 118,000

South East 231,500 220,,000 200,000 194,500

. London 360,000 350,000 300,000 279,500

East 182,500 178,500 175,000 168,500-South West 197,000 195,000 180,000 170,000

Wales 140,OO{) 135,000 130,000 125,000

West Midlands 154,500 152,000 147,500 139,000

Yorkshire & The Humber ' 145,500 148,500 138,000 128,000

North West 139,000 136,000 128,000 120,500,.

EastMidlandsv

143,000 145,SOq 138,000 133,000

National average 184,000 178,000 171,000 160,000

Chapter 3

Page 44: Succeeding in the Biomedical Admissions Test (BMAT): A practical guide to ensure you are fully

37BPP~LEARNINGMEDIA 'V

If the house value increases as the regional average did, howmuch should they expect to get for their house?

Example question 2A couple bought a house in Wales in 2005 for £200,000.In2008they wanted to sell the house.

So the region with the biggest increase is London, Answer A.

Region 2007 2006 Increase (£) Increase asafraction

South East 220,000 200,000 20,000 1/10

London 350,000 300..000 50,000 1/6

East 178,500 175,000 3,500 1/50

South West 195,000 180,000 15,000 1/12

Wales 135,000 130,000 5,000 1/26

We then repeat this for the other regions:

As you are not allowed a calculator it's easier and quicker toleave the difference as a fraction.

£50,000/£300,000= 1/6 increase

We then express this increase as a fraction of 2006values:

£350,000- £300,000= £50,000

Price increase from 2006 to 2007was

eg for London:

We then calculate the difference as a percentage increase of2006 figures.

Section 1: Aptitude and Skills

Page 45: Succeeding in the Biomedical Admissions Test (BMAT): A practical guide to ensure you are fully

BPP~LEARNING MEDIA W

38

Example question 3What is the difference in the value of the house in 2008 if thehouse value was estimated using the national average insteadof the regional average?

So the correct answer is A.

£200..;. 25 = 8

£200,000 -i- 25 = £8,000

3/25 = £8,000 x 3 = £24,000

£24,000 + £200,000 = £224,000

We first calculate 1/25 of the value

The couple paid £200,000, so we need to calculate 3/25 of thisand add it to the value.

15,000 -i- 125,000 = 15/125 which can be reduced to 3/25

We then calculate the increase as a fraction:

£140,000 - £125,000 = £15,000

AnswerFirst we need to calculate the average increase in house pricesfrom 2005 to 2008 in Wales.

A £224,000B £220,000C £215,000D £237,000E £210,000

Chapter 3

Page 46: Succeeding in the Biomedical Admissions Test (BMAT): A practical guide to ensure you are fully

39BPP~LEARNINGMEDIAW

If the property were valued using the national average it wouldbe worth £6,000more than if it was valued using the regionalaverage. The answer is B.

£230,000- £224, 000 = £6,000

We then calculate the difference between the two estimates:

1/10 of £200,000= £20,0001/20 of £200,000= £10,0003/20 of £200,000= £30,000£30,000+ £200,000= £230,000

The couple paid £200,000.Weneed to calculate 3/20 of this andadd it to the value.

24,000+ 160,000= 24/160 which can be reduced to 3/20

We then calculate the percentage increase:

£184,000- £160,000= £24,000

AnswerWehave already worked out the estimated value of the houseusing the regional averages in Wales. We need to do the samefor the national average:

A £24,000moreB £6,000moreC £9,000moreD £6,000lessE £9,000less

Section 1: Aptitude and Skills

Page 47: Succeeding in the Biomedical Admissions Test (BMAT): A practical guide to ensure you are fully

BPP~LEARNING MEDIA W

40

AnswerThe question asks what volume of air was breathed in a minute.We can determine from the graph that at 2%CO2 a mean volumeof 800cm3 is taken in per breath. The graph also tells us thatat 2% CO2 a mean of 14 breaths are performed per minute. Soto calculate how much air is breathed in a minute we need tomultiply the volume of air taken in on one breath by the numberof breaths taken in a minute.

A 800cm3B l1,200cm3C 700cm3D 12,OOOcm3

What was the average volume of air breathed in one minute ata concentration of 2% CO2?

Concentration of CO2 In air (%)65432o

!25 ~·e20]

"lii.,15li

30

c 10.8e"5 cc:lo -I-----I--+--+----t-----f----+ 0 ::E

2500

__ Mean Volume of one breathcm3

-<\>-Mean number of breaths/min

-"'e 2000~s::1500li'0~ 1000

"gI: 500....::E

The graph below shows the effect of an increasing carbon dioxideconcentration on breathing.

Example question 4Questions 4, 5 and 6 refer to the graph below:

Chapter 3

Page 48: Succeeding in the Biomedical Admissions Test (BMAT): A practical guide to ensure you are fully

41

4%CO2 = 1,300x 15 = 19,500cm3

5%CO2 = 1,900x 16 = 30,400cm3

6%CO2 = 2,100 x 27 = 56,700cm3

BPP~LEARNING MEDIA W

So we can calculate the volume of air breathed in per minutefor 4%, 5% and 6% CO2 by multiplying the number of breathsin a minute by the average volume of each breath, and thencalculate the differences:

It is reasonable to say that the greatest increase is either going tobe between 4-5%CO2or 5-6% COu aswe can see from the graphthat the mean volume of one breath does not increase a greatdeal between 1 and 3%CO2 nor does the number of breaths perminute (when compared to the increase seen at concentrationsof 4% and above), so this eliminates option A.

This question is similar to the one above. A common mistake tomake is to look at the graph and assume the greatest increase isbetween 4 and 5%COu where the gradient of the mean volumeof one breath is greatest. However, the question asks for greatestincrease in the total air breathed in per minute - so we also needto take the number of breaths per minute into consideration.

A 2-3% CO2

B 3-4% CO2C 4-5% CO2D 5-6% CO2

Example question 5Between which concentrations of CO2 is the greatest increase inthe total volume of air breathed in per minute seen?

Therefore the correct answer is B.

= 14 x 800= l1,200cm3 per minute

Section 1: Aptitude and Skills

Page 49: Succeeding in the Biomedical Admissions Test (BMAT): A practical guide to ensure you are fully

LEARNING~E~~ @42

A is the correct answer.

We then add this to 2,100 to give a 15% increase = 2,100 + 315= 2,415cm3

2,100 .;-10 = 210

So 10% = 210

5% = 210 .;-2 = 105

So 15% = 210 + 105 = 315

AnswerThis is just a simple percentage calculation. At 6% C022,100cm3

of air was breathed in each breath. So at 7% CO2 2,100 + 15%was breathed in.

A 2,415cm3B 2,115cm3C 2,205cm3

D l,785cm3

Example question 6At a concentration of 7% CO2 (data not shown), the mean volumeof one breath was 15% greater than that at 6% CO2, What wasthe mean volume of one breath at 7% CO2?

Therefore, the correct answer is D.

The greatest increase is seen between 5 and 6% CO2,

Differences between 5% and 6% = 56,700 - 30,400 = 26,300cm3

Differences between 4% and 5% = 30,400 - 19,500 = 10,900cm3

Chapter 3

Page 50: Succeeding in the Biomedical Admissions Test (BMAT): A practical guide to ensure you are fully

43BPP~lEARNING MEDIA W

Another factor contributing to such a slow response fromdevelopedcountriestoprovideassistanceis thefactthat the aboveincidences occur on a global scale and in such large numbers.Encouragingthose that canmake a differenceto faceup to theirresponsibilities is no mean feat and requires commitment tochangeboth froma personalperspectiveand a government one.Futurehealth professionalsare one community that canmake apositivedifferenceto the state of the world's health in the formofboth acting as rolemodelswho lead carbonneutral lifestylesand canvassing positive action in the name of improving thehealth of the planet.

Virtually all of the countries that are most susceptible to theabove risks do not have the resources and infrastructure toeffectivelyaddress these hazards, whilst at the same time FirstWorld countries have been lethargic in offering their supportto those that need it.

The increased incidenceof disease causing entities, the lack ofsafe drinking water, susceptibility to serious flooding and theconstantly reducing number of durable crops which providestaple foods are having an impact on a global scale.

The health of an individual is significantly influenced by theenvironmentaround them and the state ofPlanet Earth's healthis often considered to be one of the largest worries facing thehealth of the human race in modern times. The distressedcallingsof what used to be considered 'ranting on a soap box'are no longer discounted by the masses and the relationshipbetweenadverse climatechangeand someone'shealth is clearlydemonstrated by leading authorities such as the WorldHealthOrganisation.

Example question 7 -10Examplequestions 7 to 10refer to the followingp.assage.

Section 1: Aptitude and Skills

Page 51: Succeeding in the Biomedical Admissions Test (BMAT): A practical guide to ensure you are fully

BPP~LEARNING MEDIA W

44

Example question 7If the UK's total annual carbon emissions are 300,000,000milliontonnes, how many tonnes of carbon emissions are produced bythe production of domestic energy?

Identifying and taking responsibility for the impact that mankindhas on the environment and formulating positive steps to counterthis is becoming increasingly important and is the only way thatthe world's health will begin to noticeably improve.

Initiatives such as utilising public transport, ensuring domesticappliances are switched off when not in use, or avoiding airtravel through opting to spend holidays at home would havea notable effect.

A recent newspaper article raised the need for medicalprofessionals to endeavour to determine the ideal average amountof carbon an individual should produce if mankind is to standany chance of reducing and ultimately preventing' catastrophiceffects' on the planet. Based on the premise that each person musthave an equal annual 'personal carbon allowance', this can becalculated at 1 tonne of carbon per person per year. A circulatedquestionnaire amongst medical professionals canvassing carbonusage determined a large proportion of the medical professionalscreated carbon emissions significantly more than 1 tonne peryear. The general consensus was that the above average figurewas unobtainable, which prompts the call for more proactiveand creative approaches to tackling the health of the planet.Thirty-three per cent of the UK's carbon emissions are createdby domestic energy production and a further 26 per cent isattributable to transport emissions. This therefore suggests thatthere is huge potential for reducing these emissions throughrelatively small changes to our everyday lives.

Chapter 3

Page 52: Succeeding in the Biomedical Admissions Test (BMAT): A practical guide to ensure you are fully

45

Option B is not inferred by the passage as it refers to countriesbeingsusceptibletorisks due to lackofresourcesand infrastructureto effectivelyaddress these hazards.

AnswerOption A is not inferred by the passage as it refers to utilisingpublic transport, ensuring domestic appliances are switched offwhen not in use or avoiding air travel through opting to spendholidays at home.

A Initiatives such as recycling and lower emission cars arehaving a notable effecton reducing global warming

B The susceptibilityof countries to natural hazards is causedprimarily by a lack of money

C The effects of natural hazards on developing countrieshave been exacerbatedby the slow response ofFirstWorldcountries in providing support

D Reducing carbon emissionswill not make much differenceto the health of the planet

Example question 8Which statement is correctly inferred by the passage:

33%of 300,000,000million1%=300,000,000million + 100= 3,000,000million33%= 3,000,000million x 33 = 99,000,000million tonnes

AnswerThe passage tells us that 33% of the UK's carbon emissionsis created by the production of domestic energy. If the totalemissions for the UK is 300,000,000million we simply need tocalculate33%of this.

Section 1: Aptitude and Skills

Page 53: Succeeding in the Biomedical Admissions Test (BMAT): A practical guide to ensure you are fully

BPP~LeARNING MEOlA W

46

AnswerThe correct answer is option A as this is stated in the penultimateparagraph.

A Ensuring domestic appliances are switched offB Avoiding public transportC Utilising air travelD Implementing recycling measures

Example question 10Which one or more of the options below will have a positiveeffect on reducing carbon emissions?

AnswerThe correct answer is option C

A MagazineB Website articleC Newspapero Tabloid

Example question 9In what entity was the article relating to medical professionalspublished?

The correct answer is therefore option C.

Option D is not inferred by the passage as in the final paragraphit states that formulating positive steps is the only way theworld's health will begin to noticeably improve.

Option C is inferred by the passage as it states 'whilst at thesame time First World countries have been lethargic in offeringtheir support to those that need it.'

Chapter 3

Page 54: Succeeding in the Biomedical Admissions Test (BMAT): A practical guide to ensure you are fully

47BPP~LEARNING MEDIA W

• Refresh your memory by working through your GCSEmaths books to ensure you are familiar with thefollowing:

Addition and subtractionMultiplication and divisionFractions and percentagesConverting fractions, decimals and percentagesDetermining mode, mean and median averagesAlgebraDecimalsDistance, time and speed trianglesCalculating area and perimetersAnalysing charts, bar charts, pie charts, frequencytables etcSquare and cubed numbers

• Work through the questions systematically. You mayfind that a question refers to your previous answer(s).

• Workout all your calculations on the note paper provided.If there are enol'Syou may be able to determine from yourrough workings at what point you made a mistake.

• Go through the practice mock papers and identify yourstrengths and weaknesses early so you can improveon your weaknesses. For example, you may be better atcompleting algebra equations rather than fractions.

• When answering questions which involve 'humans',remember to calculate your final answer to the nearestwhole number as people cannot be represented as adecimal or a fraction! This may be an important pointto take note of when converting percentages to actualnumbers.

Top tips for succeeding in Section 1

Section 1: Aptitude and Skills

Page 55: Succeeding in the Biomedical Admissions Test (BMAT): A practical guide to ensure you are fully

BPP~LEARNING MEDIA W

48

"•• ~'" _"W"".__ "_" "".""' .""~" '""•.•"""""."""._." •• _ •."'.'" •• ~' _~~.," _ •• ' ~. ~

• One major pitfall is to select an option which you think is ·nearest to the answer, Often you will find that the majorityof the options are very dose to each other and may differin terms of decimal points or a single digit which iseither added or removed: Therefore it is very importantto evaluate the answer options very carefully. ~

• Always answer questions in the correct metric units. Forexample" a question may ask you to calculate somethingin centimetres bu t then give your final answer inmetres. Therefore it is Important to read each item veryclosely.

• Some algebra questions may require you to calculate thevalue of 'x'. Often this will be x on its own or sometimesthe answer may require you to find J!> ox x2• Therefore itis always important to'Iook at how the questions requireyou to give your final answer.

• Remember to time yourself as you complete the mock test.Thiswill improve your time management skills, ensuringyou have adequate time to answer every question.

• If you really axe unsure about a question eliminate theobvious wrong answers and then make a calculatedguess. You will not be penalise€l for getting an answerwrong. Aguess means that you have a 25 per cent chanceof getting the mark, so it is better to guess than to leavethe question blank!

Chapter 3

Page 56: Succeeding in the Biomedical Admissions Test (BMAT): A practical guide to ensure you are fully

( , . .4:>- <1:.1'/

Chapter 4Section 2:

.~."_ <_. •

Scientlfic ~• "? - . •

Knowledge and ..Applications

Page 57: Succeeding in the Biomedical Admissions Test (BMAT): A practical guide to ensure you are fully

BPP~LEARNING MEDIA \Q

50

Preparing for Section 2Believe it or not, this section looks a lot worse than it actuallyis. The idea that the questions will be drawn from an enormousnumber of topics within Maths and the Sciences may seem

Bear in mind that, as in Section I, you are not allowed to use acalculator to answer this section. As such, you should alwaystry to write your workings down. Youwon't get any marks foryour workings, but under pressure even basic mathematicalquestions will become more difficult, so you can only benefitfrom using the spare pages provided.

Section2 of the BMATwill examine your ability to apply relativelybasic scientific and mathematical knowledge to given scenarios.You will have 30 minutes to answer 27 questions on subjectsdrawn from GCSEDouble Science (including Biology,Chemistryand Physics) and Mathematics. The difficulty of the questionswill be anywhere up to higher level. Each question is worth onemark and you should expect to see between six to eight Biologyquestions, six to eight Chemistry questions, six to eight Physicsquestions and five to seven Mathematics questions.

Overview of Section 2AJthough your A level results will give your universities the bestindication of your level of knowledge, they do not show howgood you are at using that knowledge in the right way. Being ableto remember and recite scientific or mathematical facts underexam conditions isn't enough to succeed as a doctor. Being ableto apply your knowledge, including knowledge that you maynot have accessed or revisited in a long time, is key to passingyour Medical or Veterinary degree and becoming a skilled andknowledgeable doctor or vet.

Section 2: Scientific Knowledge andApplications

Chapter 4

Page 58: Succeeding in the Biomedical Admissions Test (BMAT): A practical guide to ensure you are fully

51BPP~LEARNING MEDIA W

Just like the rest of the BMAT,this section isn't designed to catchyou out. Youwon't be asked anything that you couldn't possiblyalready know, and since you'll most likely be studying a rangeof Sciences at A level anyway, you will probably have revisitedmany of the potential topics already. Remind yourself of someof the more common formulas and principles and you shouldbe able to cover most of the relevant knowledge to succeed inthis section.

Past papers of the BMAT are also available to downloadfrom the BMATwebsite directly. Visit www.admissionstests.cambridgeassessment.org.uk/ adt/ to ensure you are as fullyprepared as possible.

They will give you a good idea of the range of questions youcould be asked. Have a look through some GCSEpast papers,which are available for download from the exam boards' websitesor you may still have some of these lying around. Or brush upon your knowledge using GCSEtextbooks. Whichever way youchoose to revise, it's important to refresh your knowledge.

• AQA: www.aqa.org.uk• Edexcel: www.edexcel.org.uk• OCR:www.ocr.org.uk

Essentially, this leaves questions relating to Human Biology,andbasic Maths, Physics and Chemistry. You can look at the GCSEcurricula on each of the major examinations boards' websites:

horrifying, but in reality it's not too difficult to work out whichareas to prepare. This particular section will test your ability toapply your Scientificand Mathematical knowledge encounteredin non-specialist secondary science and maths courses up to andincluding national curriculum key stage 4. More informationabout this can be obtained at www.education.gov.uk/schools/teachingandlearning / curriculum / secondary

Section 2: Scientific Knowledge and Applications

Page 59: Succeeding in the Biomedical Admissions Test (BMAT): A practical guide to ensure you are fully

52

The correct answer is A, as statements I, 2 and 4 are true.

AnswerOption 1 True, a virus has a protein coat. It is referred to as a

capsidOption 2 True, cervical cancer has been linked to Human

Papilloma Virus. Girls are currently being vaccinatedagainst this

Option 3 False, TB is a caused by a bacterial infectionOption 4 True, a common example is the Herpes Simplex Virus

(cold sores), where the virus can lie latent in its hostDNA, meaning you can never be cured

Option 5 False, it is bacteria that exist naturally in the body, suchas the bacteria in our gut involved in the digestionof food.

A 1,2 and 4B 1,2, and 5C 1,3, and 5D 1 aloneEland 2

1 They have a protein coat2 They can cause cancer3 They can cause TB4 Their DNA can integrate into human DNA5 They exist naturally in the body

Section 2: worked examplesExample question 1: BiologyStudy the following statements about viruses. Which statementsare true?

Chapter 4

Page 60: Succeeding in the Biomedical Admissions Test (BMAT): A practical guide to ensure you are fully

53BPP~LEARNING MEDIA W

Then we calculate the volume of the small cone:

Volume = 73 1t r2h= 73 x 3 x 32 x S

= 4Scm3

Volume = 73 1t r2h= 73 x 3 X 62 X 10= 360cm3

We take 11 to be 3 (as you are not allowed calculators)

AnswerFirst we calculate the volume of the whole cone:

EuIJ')

Example question 2: MathsAfrustrum of a cone is formed by cutting the top off a cone. Theoriginal cone has a base radius of 6cm and height of 10em.Thepart of the cone removed has a base radius of 3cm and heightof Scm.What is the volume of the frustrum? Give your answerto the nearest crrr'.

Section 2: Scientific Knowledge and Applications

Page 61: Succeeding in the Biomedical Admissions Test (BMAT): A practical guide to ensure you are fully

BPP~LEARNING MEDIA W

54

Speed = distance (m) -7- time (s)

= 1200 -7- 60

= 20 m/s

We then need to calculate the speed of the car in m/ s

Mass = weight in Newtons -7- gravitational field strength (N /kg)

= 8,000 -7- 10

= 800kg

AnswerWe first need to calculate the mass of the car:

A 320,OOOJB 80,OOOJC 160,OOOJD 160,OOOkJE 320,OOOkJ

Kinetic energy 0) = 1hmass (kg) x speed/ (m/ s)

Example question 3: PhysicsA car with a weight of 8,OOON,travels 1,200 metres in a minute.Use the following equation to calculate the kinetic energy of thecar. (Take the gravitational field strength as 10 N /kg).

= 360cm3 - 4Scm3

= 31Scm3

We then subtract the volume of the small cone from that of theoriginal cone to find the volume of the frustrum:

Chapter 4

Page 62: Succeeding in the Biomedical Admissions Test (BMAT): A practical guide to ensure you are fully

55

So the correct answers are A, C and D.

InBand E, copper is oxidised, as copper has lost two electrons(Cu ~ Cu2+ + Ze")

A reduction reaction can also be defined as losing oxygen, asin C and D. The copper in CuO, is Cu2+ and upon reactionwith carbon or hydrogen is reduced to Cu. So C and D are alsoreduction reactions of copper.

Reduction is where an element gains electrons. InA, coppergains 2 electrons as it goes from Cu2+ to Cu (Cu2++ Ze" ~ Cu),so A is a reduction.

AnswerWhen faced with an oxidation / reduction question rememberthe acronym OILRIG: Oxidation Is Loss of electrons, ReductionIs Gain of electrons.

A Fe + Cu2+Fe2++ CuB 2Cu + O2 2CU02C 2CuO + C 2Cu + CO2D CuO + H2 Cu + H20E Cu + 2Ag+ Cu2+ + 2Ag

Example question 4: ChemistryInwhich of the following reactions is copper reduced?

The correct answer is C.

So kinetic energy of the car = 1h800 x 202

= 400 x 400

= 160,OOOJ

Section 2: Scientific Knowledge and Applications

Page 63: Succeeding in the Biomedical Admissions Test (BMAT): A practical guide to ensure you are fully

• 0<...>' t;..J"

Section 3:Writing Task

Chapter 5

Page 64: Succeeding in the Biomedical Admissions Test (BMAT): A practical guide to ensure you are fully

58

Can you approach an argument from differentperspectives?

Unlike your Alevel essays, you will not be required to demonstrateresearch or knowledge of specific facts or concepts. You will bepresenting your own views, opinions and arguments based onwhat you think about the issues raised, so the emphasis is firmlyon the way in which you approach the question. You shouldconsider the following:

• Understanding - what do you understand the statement tomean?

• Argument - can you provide an argument to oppose thestatement?

• Resolution - can you resolve the situation or give examples todemonstrate why the statement mayor may not be true?

This section of the test requires you to write a short essay (limitedto one side ofA4) based on a statement or quote. Youwill have 30minutes inwhich to complete this section. Youwill be providedwith four statements to choose from. Each will present an ideawhich you will have to assess, understand and critically argue,both for and against. The basic format for each statement willfollow a similar pattern:

During your time at university, you will come across manynew or complex concepts and ideas. Youmay agree with someof them, and you may disagree with others, but the importantpoint is how well you understand what you are reading andhow well you can communicate your own views, opinions orideas to your fellow peers. As a doctor or vet, this skill will beessential in both your day-to-day clinical work, as well as inactivities such as teaching, research and presentations.

Overview of Section 3

Section 3: Writing Task

Chapter 5

Page 65: Succeeding in the Biomedical Admissions Test (BMAT): A practical guide to ensure you are fully

59BPP~LEARNING MEDIA W

Finally,don't wait until aweekbeforethe examto start franticallychecking out journals or waking up at 7am every day to getthe morning newspaper. Start your reading a couple of months

Youshould also be prepared to read around your studies; readjournals, newspapers and medical websites to keep up-to-datewith current clinical, legal or ethical issues in the world ofmedicine.Not onlywill this help with your current studies andgiveyou something to include on your personal statement, youwill hopefully acquire a good range of knowledge to draw onwhen answering the question. Make notes of your research,and have an opinion! Youwon't get great marks just for recitingverbatim the recent debate over stem cell research or repeatingan article you read on human cloning. Understand how theissues you're reading about affect the real world, and comeupwith arguments as to why you do and don't agree with them.Even if you don't have the chanceto use these specificexampleswithin the exam, you'll at least get some practice in debatingthesekinds ofissues and putting acrossyour consideredopinionin a clear and well-thought out way.

Preparing for Section 3Itmay seemat first that this is a very difficulttask to prepare for.It's true that you can't just learn to recite a whole load of factsand theories, but you can learn how to approach the questions inthe right way.There are plenty of textbooks that cover how tothink criticallyand formulate arguments; go to the library andread through a few to get an idea of the style and techniquerequired for a good answer.

Onceyou have grasped these concepts,you should be in amuchbetter position to attempt this sectionof the BMAT.

• Can you clearly communicate your views and ideas?• Can you resolve conflictbetween opposing arguments?• Can you illustrate your argument with well-thought

examples?

Section 3: Writing Task

Page 66: Succeeding in the Biomedical Admissions Test (BMAT): A practical guide to ensure you are fully

60

Choosing a question!There won't be a 'right' or 'wrong' statement to choose; eachstatement will have its own complexities and pitfalls, so noone statement will be any more difficult than the other. W1nchstatement you choose will be entirely up to you, so ensure youtake a minute or two to read through the options and choose the

Plan ahead!The fact that you have a limited space and time in which tocompose your essay means that you will have to focus your ideasquickly and effectively.As with any other essay, the best way todo this is to plan ahead; space will be provided in the questionpaper where you can make notes - use it! Get some key ideaswritten down and know what you're going to say so you don'thave to start writing blindly. You'll have 30 minutes in total;since you're only writing 300-350 words, there shouldn't be aproblem spending five to ten minutes ordering your thoughtscoherently.

Tips for writing essaysJust as in Sections 1 and 2, you will be provided with a paperanswer sheet in which to write your answer. The answer sheetis one side of A4; your answer can be no longer than the spaceprovided, as no further answer sheets will be available. Thelength of your answer will obviously depend on the size ofyour handwriting, but you should be aiming to write around300-350 words. You should also ensure that your writing is asneat as possible.

before the exam and try to keep it going. Build up a good setof notes and resources and you'll have plenty to write aboutin the exam. The university may also ask you to elaborate onanything you discuss during the interview, so you'll definitelybenefit from having an in-depth understanding of what you'rewriting about.

Chapter5

Page 67: Succeeding in the Biomedical Admissions Test (BMAT): A practical guide to ensure you are fully

61BPP~LEARNING MEDIA '"

• Build your argument in a logical way; each sentence shouldfollow into the next until you reach your conclusion

• Don't waste your word count - every sentence should berelevant to the question

• Have a conclusion! Don't let the essay just fade away intonothing right at the end. Youdon't have many words to playwith, but your argument should certainly reach a satisfyingconclusion.

Get to the point!In a normal essay you would be expected to refer to your researchand explain, in detail, how you came to your conclusion. TheBMAT essay is more focused on reason and argument thanpresenting hard evidence. Sinceyou won't have to worry aboutciting quotes or including insightful research, you can movequickly into addressing the question at hand. Bear in mind thefollowing:

Read the question!You've probably heard this a hundred times already regardingexams, but it's an important point to bear in mind. The BMATisn't designed to trick you or catch you out. The questions aresimple and to the point - all you need to do is make sure thatyou are answering the question that's in front of you. Eachquestion will have a set of prompts, which are there to helpyou in setting your train of thought in the right direction. Theywill also provide you with a rough idea of how to structureyour argument.

statement you think you could fully answer in the best possiblemanner. The best approach to choosing a statement is to trustyour gut instinct - if you look at a particular statement andimmediately think 'I could write something about that', then itmay be the best one to choose.

Section 3: Writing Task

Page 68: Succeeding in the Biomedical Admissions Test (BMAT): A practical guide to ensure you are fully

BPP~LEARNING MEDIA W

62

We have included a summary of the scale below for quickreference.

www.admissionstests.cambridgeassessment.org.uk/ adt/bmat/practice

Section 3: mark schemeWhen examining your answer, the assessors will mark youressay according to a set scale. The original scoring scale can befound at:

Check for errorsWhere possible, you should leave a minute or two at the end ofthis section to go back and check through your essay for errors.When you're writing against the clock you're bound to makeone or two spelling or grammatical errors, so it is worthwhileat the end to go back and check for any obvious mistakes. Asdiscussed overleaf, you'll be marked on your clarity of languageas well as content, so eliminating any grammatical errors canonly work in your favour.

Be coherent!You've read the options, chosen a question and scribbled downan essay plan to get a good start! All you need to do now is totransfer the words from your head, via your pen, to the answerpaper. Sounds easy enough, but under pressure it's tempting totry and overdo it to squeeze a few more marks. Being able to writebeautiful, flowing prose will certainly be an advantage, but you'renot expected to write an essay of Shakespearian proportions.Too much flowery prose may obscure what was otherwise aperfectly good point. Avoid using complex terminology unlessyou're absolutely sure of what it means, and avoid unnecessarilylong words - don't say 'hippopotomonstrosesquipedaliophobia'when what you mean is 'fear of long words'.

Chapter 5

Page 69: Succeeding in the Biomedical Admissions Test (BMAT): A practical guide to ensure you are fully

63

Quality of English marking criteriaMarkers of the essay must consider whether the candidate hasused the English language in a concise,convincing and correctmanner.

Score DescriptioI')

0 The answer is completely irrelevant to the question asked

1 The points ghlen in the essay have some retation to the -qu:estionr asked but does not answer the question in the way that is beingask&dand rs unstructured

2 The answer given deals with the majority of the points asked inthe question and is presented in a relatively clear and succinctmanner. The answer may contain contradictions in the way theanswer iswritten and the points givenfor and against the argumentmay not be convincing

3 Arelativelywell constructed response that answers aUaspects ofthe questien asked. The candidate will illustrate their argumentwith a broad use of the ettldeRce prov;idedand with a good senseof clarity. There may however be some points in the dIsCUSSionprovided in ~.rms of points against the argument that lackthrustor a particular aspect of the answer tacks depth

4 Astrong answer that exhibitsa smallnumber of negatives.Togetherwith presenting a well constructed response that answers allaspects of the question asked, the debate iswell-evidenced usingthe material provided. The points for and against are presentedin a considered and balanced manner

,5 Ariillativelyhighquality response that demonstrates few negatives.

iOgl'ltherwith presentilliga weHconstructed response that answersaUaspects of the question asked, excellent use of the evidenceprovided will have been made to pr0vi(le a convincing argument

; for ami aga:l'nSt.Paints are presented in an extremely clear and ,logical maRner resulting fA a co;ncise and highly convincing·conoluslon

Section 3: Writing Task

Page 70: Succeeding in the Biomedical Admissions Test (BMAT): A practical guide to ensure you are fully

64

Reference:www.admissionstests.cambridgeassessment.org.uk/ adt IdigitalAssets I 126401_Explanation_o£_BMAT_Results_20ll. pdf

Band E - weak use of EnglishThe answer does not flow coherently and is confusing at times.Poor use of vocabulary with significant errors in grammar,spelling and punctuation.

Band C - reasonably dear use of EnglishThe answer is relatively coherent and not difficult to read butthere may be some poor use of English. Sentences presented ina simple structure with a reasonable range of vocabulary used.Use of grammar is reasonable with acceptable spelling andpunctuation but some errors are present.

Band A - good use of EnglishThe answer flows coherently and is easy to read with wellconstructed sentences. The candidate uses a wide range ofvocabulary with good quality grammar, spelling and punctuationwith minimal errors.

Chapter 5

Page 71: Succeeding in the Biomedical Admissions Test (BMAT): A practical guide to ensure you are fully

65BPP~LEARNING MEDIA W

Note the key words -'what do you think'. This is an easy way tostart writing the essay. Just write about what you think itmeans- no tricks, no surprises, just write about your understandingof the statement.

What do you think the author means by this statement?

In a 'unified' essay, all of your arguments should support youressay. This may sound obvious, but it means that you need todefine exactly what you are writing about beforehand, or youressay will quickly veer away from the central point.

Write a unified essay

Okay, now let's break the question down and look at eachpart:

Towhat extent is it important to understand the personalqualities of your patients?

".,.....-~"-:w;r,_,, ......"'" ""(ffl?-'f' ~,.--=~ oor-:'~~~ '''''~I*>·...··..·~t'''''·~ ....." ~. :<'.a"'~

1 J:t is more important to know what sort of person ha'Sa disease than fo know what sort of disease a personka ~

(Hippocrates)

Write a unified essay in which you address the I£.oJlow:ing: 1What do you think the author ~ .... by tI.is statement? ICan you sug$est examples where Itmay be more nnportantto understand the disease than the person?

Section 3: worked exampleLet's have a look at an example question from Section 3:

Section 3: Writing Task

Page 72: Succeeding in the Biomedical Admissions Test (BMAT): A practical guide to ensure you are fully

BPP~LEARNING MEDIA W

66

Notes:

lit is more important to know what sort of person has adisease than to know what sort of disease a person has'

Think about the statement presented and make some notes inthe space below - how would you go about answering thisquestion? Then see if your ideas match up to our example noteson the next page.

Any essay needs a good conclusion to draw all of the pointstogether. Now you've discussed what you think the statementmeans (and doesn't mean), you need to resolve your opposingarguments to explore what the statement is really addressing. Thiswill be vital to presenting a logical and sensible conclusion.

To what extent is it important to understand the personal qualitiesof your patients?

Simply agreeing with an idea or concept won't be enough toenable you to formulate a strong argument. Youneed tobe able tolook at a concept from all angles and present a counter-proposal,using different points of view to construct a well-rounded andconsidered argument.

Can you suggest examples where it may bemore important to understandthe disease than the person?

Chapter 5

Page 73: Succeeding in the Biomedical Admissions Test (BMAT): A practical guide to ensure you are fully

67BPP~LEARNING MEDIA W

For example, you could simply write notes or a basic flow chartto map out your answer.

There are a number of ways you could plan your answer. Somepeople may prefer to write a few sparse notes or keywords tolink together the different aspects of their arguments. Others mayprefer to draw diagrams to visualise their answers. Try out a fewdifferent techniques and see which way you find easiest.

Thesimplest approach to planning your answer is to use the orderofquestionsprovided. Eachsub-questionwithin the main questionprovides a helpful' anchor' for you to base your arguments on.You don't necessarily have to follow this approach rigidly, butunder timed exam conditions it's a good idea to follow the moststraightforward process to writing your answer.

Now let's think about what this statement could mean. Don'tforget there will be space on the question paper to plan youranswer, so use as much of it as you need.

'It is more important to know what sort of person has a diseasethan to know what sort of disease a person has'

Planning your answerLet's start making some notes. The first step is to remind ourselvesof the statement:

Section 3: Writing Task

Page 74: Succeeding in the Biomedical Admissions Test (BMAT): A practical guide to ensure you are fully

BPP~LEARNING MEDIA '4Q

68

/Not necessarily - might help to find out how they gotit but not how to treat it - this may stop re-infectionor save the lives of others by eliminating the source ofinfection

Surely it's always important? eg if a patient has avirus - does knowing the patient well help to treat theillness?

When is it more importan t to understand the illness?

/ Could have on effect on patient heaithltreatmeni

Understanding who they are/where they arefrom

It's more important for a doctor to know his patients personallythan to know what is wrong with them

t

What does the statement "nean?

Chapter 5

Page 75: Succeeding in the Biomedical Admissions Test (BMAT): A practical guide to ensure you are fully

69

~ .". "'~'t!~~~~"~~'~'~"'~ ~"""~""'.>';;.:~'Y.t' '!W:'f1

The sta.tement by Hippocrates describes something thilt" in modem ~medicine, 'Wou14 be called a 'hoUsleicapproach',' He is stflFing that ,simply understanding what is wrorl:gwith a patient is not sufficient 1to treat them; Ull't:ierstanaing {if patient's personal situation is just Jas impartanl to reaching a diagnosis: do they ha'lJeallergies tomedicati-on? Do their living conliitio;ns 01' lifestyle have an effect onthe illness? M.anagiftg a chrOiuic illness, such as diabetes, is mucheasier if fhe doctor understands tlu! patient's diet" alcohol intake,level of exercise, etc. Considering all of these points will help the I

doctor to reach a more accurate diagnos.is.

Example answerNow you've thought about the question and made a few notes,let's examine a possible answer:

Whichevermethod you prefer,make sure you do make time toplan your answer out - don't get halfway through a questionand realiseyou don't have anything to say!

Need to use practical and personal skills together/ for best effect

~It is important but not necessarily more important than understandingthe illness - without agood practical/medical understanding, the doctorcannot provide the necessary medical care

Towhat extent is it important to understand the personal qualities ofyour patients?

Section 3: Writing Task

Page 76: Succeeding in the Biomedical Admissions Test (BMAT): A practical guide to ensure you are fully

70

Paragraph 1file 'st;te;;;;;tby Hippocraies describes sOI;;~'tj;i;;gthat, in modem 'medicine, 'Would be called a 'holistic approach'. He is stating that :simpLy understanding what is wrollg with a patient is not sufficient'to treat them: understanding a patient's personal situation is just!as important to reaching a diagnosis: do they have allergies to .medication? Do their lioing conditions or lifestyle have an effect on

Note that we have used the prompts from the plan to structureour answer; each paragraph corresponds to one of the smallerquestionsasked aspart of the originalquestion,whichhas helpedus to structure our answer with a solid 'beginning-middle-end'format. Let's look at each paragraph in turn and examine howwe came to this answer.

A doctor needs to be able fa understand both the patient and tiledisease they are treating and combine thie knowledge to fonn atreatment plan for iluit particular patient. Possessing a good generalmedical knotoledge is important, but without understanding the'patient the doctor may suggest an inappropriate or ineffective courseof action. Similarly, for a doctor to know about a patient's job OT .family l~febut not understand how to cure them of their illness :would be a dangerous situation. If is not more important for a 'doctor to understand the patient, but it I::;an extremebj important .part ?Lthe_.~·~atlT1el1tprocess.

.However, it is important fo remember that every doctor must:possess relevant and up to date medical knouiledge. Understanding ~a patient's situation is irrelevant if a doctor is unable to make the !

correct diagnosis and design all appropriate treat inent plan. For 'example, while it is important to know ltou: or where a patient 1

contracted a virus, it is equally important that tile doctor is ableto idcntifiJ the virus, prescribe the correct treatment and advise thepatient on how to recover. Practical knowledge and skills are vital,and every doctor should work hard to ensure that his 07' her own:kllowledge is sufficielltly up to date at all times. !

Chapter 5

Page 77: Succeeding in the Biomedical Admissions Test (BMAT): A practical guide to ensure you are fully

71BPP telLEARNING MEDIA W

One thing to remember is not to force toomuch into your openingparagraph. Remember what you're working towards - a well­paced, flowing and balanced essay. Tackle each sub-questionone at a time and make sure that each paragraph flows into thenext until you reach your conclusion.

This may answer the question in a very basic way - the studenthas said what they think it means - but there's no real substance tothe answer.Theyhaven't displayed a good level ofunderstandingor of being able to explore the concept instead of simply recitingwhat it means. Tie the statement into something relevant, ie areal-life, applicable situation, in order to show that what you aresaying isn't just something you've made up on the spot.

The statement by Hippocrates means that it is more important tounderstand the patient than to understand what is wrong withthem.

Inthis opening statement we have detailed what we thought thestatement meant, which is exactly what we were asked to do.To illustrate our understanding, we have included examples ofwhere the statement may apply, for example, in understandingthe lifestyle of a diabetes patient. This way, instead of simplystating the facts, we are constructing a suitable response basedon how we understand the statement. Consider the following:

What do you think the author means by this statement?

the illness? Managing a chronic illness, such as diabetes, is mucheasier if the doctor understands the patient's diet, alcohol intake,level of exercise, etc. Considering all of these points will help thedoctor to read, a more accurate diagnosis.

Section 3: Writing Task

Page 78: Succeeding in the Biomedical Admissions Test (BMAT): A practical guide to ensure you are fully

72

Bear in mind that the question specifically asks for you tosuggest examples to support your argument. 1£ you don't use anexample of some kind, you will certainly sacrifice marks andlower your overall score for this section. This is why researchinga wide variety of topics as part of your exam preparation is agood idea. Having a few 'real-life' case studies to call upon willgive you some flexibility; you could use one example, or moreif necessary. Be as specific as you can, but remember to keepyour examples relevant - don't just wedge something in becauseyou remember it, as you're being scored on the coherence ofyour argument.

Here we've presented our counter-argument to the statement,which makes up the central part of our essay. Our argument isthat, although we do not disagree with the original statement,practical skills and knowledge are alsovital in diagnosing patients.In this way, we're not simply saying 'the original statement iswrong'; we're showing that we can build a good argument thattakes all sides into account, which in turn helps us to worktowards oUI conclusion in the final paragraph.

Can you suggest examples where it may bemore important to understandthe disease than the person?

~ .~ •• +"<c:' .~ •• 'r"-""<' ........~. '..,. '....~_. _, •.• ~. , "'-"o"A....... "H~ ........... _. -~ 't

Houieoer, it is important to remember that every doctor must possess 1reieoan; and up to date medical knowledge. Undm'stmudillg apatiel'/!t1s :situation is irrelevant if a dador is unable t'Q make the correct,diagnosis and design an appropriate treatment plan. For example, :while it is uuportan! to knoui how or uiaere a patient contracted a :virus, it is equally important that the doctor is able to identifij the;

, virus and prescribe the coned treatment. Similarly, for a surgeon ieperating to remove a complex tumour, anatomical kuo~()tedgeand ;praciicul skills tuouid take precedence over a personal connection lwith the patient. Practical kllQ'Wledgeand skins are vitffl, and every :doctor should W01* hard to ensure that his or her own knowledge j

. i~~s~!.f2~i~I!J~}/!~P':'?.~~.~:.~.!al!J~n:.s.:.,,", N~".~.._'__M~.

Paragraph 2

Chapter 5

Page 79: Succeeding in the Biomedical Admissions Test (BMAT): A practical guide to ensure you are fully

73BPP telLEARNING MEDIA 'U

Pay close attention to the wording of this question, particularly'To what extent'. This could equally be worded as' How far should',or 'When is it' or 'Why is it'. The question is directing you toqualify your ideas rather than just stating a simple opinion. This

Our final paragraph is where we have resolved the two counter­arguments presented in the first two paragraphs. We havepresented a balanced and logical argument by reasoning thatboth sides of the argument are equally valid and must both betaken into account when diagnosing a patient. The conclusion isa logical progression from the previous paragraphs and neatlyties together the issues already raised (remember - a unifiedessay).

To what extent is it important to understand the personal qualitiesof your patients?

Paragraph 3A doctor needs to be able to understand both the patient and thedisease they are treating and combine this knowledge to form atreatment plan for that particular patient. Possessing agood generalmedical knowledge is important, but without understanding thepatient the doctor may suggest an inappropriate or ineffective courseof action. Similarly, for a doctor to know about a patient's job orfamily Ufe but not understand how to cure them of their illnesswould be a dangerous situation. It is not more important for adoctor to understand the patient, but it is an extremely importantpart of the treatment process.

Some people will find it easier than others to come up withexamples. Not everyone is able to quote Einstein or Newton ondemand, so work within what you know. If all else fails, keepit simple; a good quality answer with a simple example willscore much better than an answer quoting word-perfect linesfrom Darwin's Origin of Species, but which makes absolutely nosense and is irrelevant to the question.

Section 3: Writing Task

Page 80: Succeeding in the Biomedical Admissions Test (BMAT): A practical guide to ensure you are fully

BPP~LEARNINGMEDIA W

74

What do you understand this statement to mean within amedical context?

1. Intuition does not in itself amount to knowledge, yet cannotbe disregarded by philosophers and psychologists.

(Corliss Lamont)

Section 3: practice questionsNow we've looked at Section 3 in detail, take a look at theseexample questions and apply the same reasoning to each to seeif you can come up with an answer. You may find it useful totime yourself. You'11find our own notes on the following pages- see how yours compare.

Are you convinced? The writer has stated a lot of opinions buthasn't gone as far as to even try to justify them. The answer mightget a couple of marks for being written in coherent English, butthe argument itself is non-existent.

A doctor should always focus on treating the physical symptoms ofillness and should place less emphasis on understanding the patient.Successfully treating the illness is far more important than gettingto know the patient or understanding their situation, and a doctorshould not waste valuable time when they should be concentratingon treating patients.

It may be, however, that you tend to agree with one point morethan the other, or that you think all of the arguments are flawed- that's fine. You can argue in either direction, since there areno right or wrong answers to these questions, as long as youjustify your point of view. The important thing is that, no matterwhat you believe, you are able to put across your views in areasonable, convincing manner. For example:

is your invitation to take the points put forward in the first twoparagraphs and to close the argument one way or another.

Chapter 5

Page 81: Succeeding in the Biomedical Admissions Test (BMAT): A practical guide to ensure you are fully

75BPP~LEARNING MEDIA W

Towhat extent is it important for doctors to promote goodhealth through patient education?

Please note in the exam y~ will have a choice of fourquestions.

Using examples, discuss where it is important for a doctorto teach patients to look after themselves, and where it maybe more important to treat the patient directly.

What does this statement mean to you?

(Thomas Edison)

3. The doctor of the future will give no medicine but willinterest his patients in the care of the human frame, indiet and in the cause and prevention of disease.

How far does progress within medicine depend on progressin the sciences?

In what instances may the field of medicine not be directlyaffected by scientific developments and when is it relianton such developments?

What is your understanding of this statement?

2. Medicine is driven by the sciences;the more we understandabout science, the better medicine will become.

How much emphasis should a clinician place on using theirinitiative when treating a patient?

Give examples to illustrate when intuition may be abeneficial influence on patient care and when it may bedetrimental.

Section 3: Writing Task

Page 82: Succeeding in the Biomedical Admissions Test (BMAT): A practical guide to ensure you are fully

BPP~LEARNING MEDIA W

76

Dependent on situation ie A&Eversus long term treatmentplan

How much emphasis should a clinician place on.using their intuitionwhen treating a patient?

• Detrimental - the doctor may be isolated and afraid to askfor help.

• Beneficial- emergency or trauma situations - for example,a patient may present with unidentified bleeding after a caraccident; in order to save their life, a surgeon may have toact without ordering the standard tests (X-rays, MRIs) andperform surgery without being fully prepared.

Give examples to illustrate when intuition may bea beneficialinfluence011. patient careand when it may be detrimental.

• Intuition is what enables doctors to question situations andfurther the field of medicine.

• Even though intuition cannot replace medical knowledgeand training, it cannot be disregarded in terms of patientcare.

• Intuition is not knowledge in that it does not necessarilydepend on proven or recognised factual information.

• Intuition and knowledge are separate concepts but are notcompletely independent from one another.

What do you understand this statement to mean within a medicalcontext?

Section 3: example answer 1Intuition does not in itself amount to knowledge, yet cannotbe disregarded by philosophers and psychologists.

(Corliss Lamont)

Chapter 5

Page 83: Succeeding in the Biomedical Admissions Test (BMAT): A practical guide to ensure you are fully

77BPPtealEARNING MEDIA 'U

• Could depend on a definition of 'the sciences'; psychologyand psychiatry could be influenced by the biological sciences(for example in linking psychopathic behavioural disordersto neurological conditions), and could also be associated withsocial sciences.

In what instances may the field of medicine not be directlyaffected by scientific developments, and when is it reliant on suchdevelopments?

• Alternatively, it could also suggest that doctors mustincorporate the philosophy of a close relationship withscience in order to further their own profession.

• The statement could suggest that the roles of doctors andscientists are separate; that scientists work in one discipline,while doctors work in the other.

• Medicine and science are inextricably linked; medicinedepends on progress in the sciences and cannot progressof its own accord.

What is your understanding of this statement?

Section 3: example answer 2Medicine is driven by the sciences; the more we understandabout science, the better medicine will become.

• Where possible intuition should be used to debate decisionswith colleagues to ensure correct decisions are made.

• Where possible facts should be established in order to makea decision, however the patient's care should be priorityabove all, and sometimes doctors do not have the luxuryof time.

Section 3: Writing Task

Page 84: Succeeding in the Biomedical Admissions Test (BMAT): A practical guide to ensure you are fully

BPP~LEARNING MEDIA W

78

(Thomas Edison)

Section 3: example answer 3The doctor of the future will give no medicine but will interesthis patients in the care of the human frame, in diet and in thecause and prevention of disease.

• Evidence-based medicine relies on the discovery of bothscientists and doctors to enable the development of newtreatments or the refinement of existing approaches.

• 'Clinical science' is closely related to medicine and drawsupon many aspects of the' classical' sciences.

• As a profession, medicine is something of a hybrid; itincorporates elements of the sciences, but also depends onother disciplines.

How far does progress within medicine depend on progress in thesciences?

• Developing countries have no access to the latest treatmentadvances.

• Inorder to treat a disease, it must be understood - eg in thetreatment of Alzheimer's, until the pathway that leads tothe build-up of protein deposits in the brain is understood,how can doctors go about treating it? In this case medicineis reliant on scientific development.

• Most advances in treatment are due to research by scientists(eg cancer treatment, heart disease and stern cell therapy); itcould be argued medicine is reliant or directly affected bythese.

• Most medical equipment is reliant on developments in science(eg MRIs, X-rays).

Chapter 5

Page 85: Succeeding in the Biomedical Admissions Test (BMAT): A practical guide to ensure you are fully

79BPP~LEARNING MEDIA W

To what extent is it important for doctors to promote good healththrough patient education?

• On the other hand, certain conditions (eating disorders andmetabolic illnesses) could lead to patients becoming obesethrough no fault of their own; regardless of diet, thesepatients require medical intervention or risk further illnessor death.

• Poor diet ultimately results in further medical complexities,such as diabetes (and all its related problems), high bloodpressure and heart conditions; thus, an obese patient whoseweight is their own responsibility could tie up resourcesand thus prevent others from receiving timely and costlytreatment.

• Current debates about obesity suggest that it is importantfor any patient to take personal responsibility for their ownactions; in most cases, obesity is related to a poor diet andlack of exercise.

Using examples, discuss where it is important for a doctor to teachpatients to look after themselves, and where it may be more importantto treat the patient directly.

• Prevention is better than a cure.

• Medicineand healthcare will eventually be the responsibilityof the patient, and that patients will only resort to doctorsin serious or unpreventable circumstances.

• Thedoctor does still carryresponsibility,but in an educationalcontext rather than through treatment and intervention.

• Progress in medicine places the emphasis firmly on thepatient, not necessarily on the doctor.

What does this statement mean to you?

Section 3: Writing Task

Page 86: Succeeding in the Biomedical Admissions Test (BMAT): A practical guide to ensure you are fully

BPP~LEARNING MEDIA W

80

Ultimately, doctors will always be required to provideinterventional treatment, regardless of how capable patientsare of looking after their own health.

However, doctors can never be completely removed fromthe equation; regardless of whether an illness is due tolifestyle or circumstance, doctors are duty bound to providecare wherever possible.

Educating patients can only prove to be a benefit in the longrun as it would take the strain off the medical infrastructureand enable doctors to treat those patients who are unableto simply help themselves.

Page 87: Succeeding in the Biomedical Admissions Test (BMAT): A practical guide to ensure you are fully

C> 0~ C> 0~ --rf ""f!- ~

C> _/Qq­

IOQ' ~-

• 0(../ <;..J"

Full "B:MATmock

Chapter 6

Page 88: Succeeding in the Biomedical Admissions Test (BMAT): A practical guide to ensure you are fully

LEARNING~E~I~ @82

Mock Section 1: Aptitude and Skills - 60minutesQuestion 1Towin a gold at the Olympic games in judo, Satoshi Ishii fromJapan had to beat six other players. How many people competedat the Olympic games in judo?

To make the most of this practice paper you can downloada sheet to record your answers by visiting www.bpp.com/freeheal thresources

Section 3: Writing Task, 1 essay question, 30 minutes

Section 2: Scientific Knowledge and Applications, 27 questions,30 minutes

Section 1: Aptitude and Skills, 35 questions, 60 minutes

Remember that you are not permitted to use calculators duringSections 1 or 2, and that your answer for Section 3 should be nolonger than one side of A4 paper. You should allow yourself nolonger than two hours, as per the real exam, to gain the greatestbenefit from this exercise. Remember the time allocations areas follows:

We recommend that you use spare sheets of A4 paper for yourworking out; in the real exam you will be able to use the blanksheets indicated as part of the test paper.

Wewould recommend that the best way to prepare for the BMATis to complete a full practice paper, under timed conditions, inorder to fully appreciate the nature of the exam you will betaking.

Full BMAT mock

Chapter 6

Page 89: Succeeding in the Biomedical Admissions Test (BMAT): A practical guide to ensure you are fully

83BPP~LEARNING MEDIA W

A A major advantage of the cream described is that it willonly need to be applied twice, saving time and money

B A specific treatment using the cream could prove a majoradvancement in the battle against antibiotic resistantbacteria

C The cream will have uses in treatments against viralinfections

D The cream is in the early stages of developmentE The bacteriophage technology used is a form of bacteria

What is inferred by the above passage?

Question 2A virus-laden cream, to prevent the spread of MRSA, could bemade available within the next two years. Scientists are at ahighly advanced stage of developing the cream, which containsa combination of various viruses which target the MRSA bug.The cream is to be applied inside the noses of hospital patients.Once in the nose the viruses will fasten themselves onto theMRSA bacteria, and infuse the MRSA bacteria with their owngenetic material. It has been proposed that viruses reproducethemselves, which suggests that repeated treatments may not benecessary, when compared to other treatments. The virus creamseems to be one of the latest ongoing examples of a resurgence ofinterest in bacteriophage viruses. Ithas been proposed that thisspecific form of treatment, involving the use of bacteriophages,may be a potential solution to the increasing problem of bacterialresistance to antibiotics.

A 60B 32C 64D 128E 124

Full BMAT mock

Page 90: Succeeding in the Biomedical Admissions Test (BMAT): A practical guide to ensure you are fully

84

Question 4A clock comprising six separately rotating number or letterpanels shows the following time. What will be the total numberof panel changes made to reach the time 01.22pm?

A 1 aloneBland 2C 2 and 3D All of the aboveE None of the above

1 Younger people are more likely to apply touniversity

2 More people are applying to university each year3 More females apply than males

Which of the following statements can be deduced from theinformation given above:

20 and under 21 to 24 Total25 to 39 40 and overAge

~500c~ 400:loE300IIIe~ 200too'E. 100C-

cc 0 -j-'-- ......---,..--l-.......__r---'----'-'---,--";,;,.;.;iI-L.,-__.___.----,

600

Question 3The graph below shows the numbers of applicants to universityfor a given period:

Chapter 6

Page 91: Succeeding in the Biomedical Admissions Test (BMAT): A practical guide to ensure you are fully

85BPP~LEARNING MEDIA 'U

Question 6Two motorbikes have a race at a track. One racer rides a Ducattiwhich has a maximum speed of 200mph, and the other rides aHonda which has a maximum speed of 150mph. The track is 10miles long. When will the Ducatti lap the Honda?

A CHD is a condition that does not need to be addressedB CHD poses a serious and increasing threat to industryC To save costs industry does not need to do anything about

CHDD The cost of informal care of CHD sufferers is insignificantE Reducing death rates in CHD sufferers will decrease efficiency

in industry

Which one of the following best describes what is inferred bythe passage?

Question 5In 2011, losses in production due to mortality and morbidityassociated with coronary heart disease cost industry over £3,100million, with around 30% of this cost specifically due to deathand 70% due to illness in those of working age, a 25% rise whencompared to costs incurred in 2002. The cost of informal care forpeople with coronary heart disease was approximately £1,250million in 2003.

A 248 rotationsB 271 rotationsC 171 rotationsD 280 rotationsE 210 rotations

A M229:o

Full BMAT mock

Page 92: Succeeding in the Biomedical Admissions Test (BMAT): A practical guide to ensure you are fully

BPP~LEARNING MEDIA W

86

BCDE

Diamond and ruby but not necessarily less precious thansapphireDiamond and sapphire but not necessarily rubySapphire, but not necessarily ruby or diamondRuby but not necessarily diamond or sapphireRuby, diamond and sapphire

A

Therefore, pearl must be less precious than:

Diamond is more precious than ruby which is more preciousthan sapphire. Emerald is less precious than ruby but moreprecious than pearl.

Question 7Matt wants to buy his wife Nicola a necklace for her birthday.He asks a jeweller for some advice. He learns from the jewellerthat:

A After 11minutesB After 12 minutesC After 13minutesDAfter 14 minutesE After 15minutes

Chapter 6

Page 93: Succeeding in the Biomedical Admissions Test (BMAT): A practical guide to ensure you are fully

87BPP~LEARNING MEDIA W

Question 9What percentage of the male population was diagnosed withprostate cancerin 2011?(Assumethat the population is equallydistributed between males and females.)

A 61,000B 6,100C 10,000D 7,200E 72,000

Question 8How many people were diagnosed with breast cancer in2011?

The population of the UK in 2011was 60million.

BowelProstateLungSkinBreast

Questions 8, 9, 10 and 11 refer to the followinginformationThe graph below shows the incidence of the most commoncancers in 2011in the UKper 100,000of the population.

Full BMAT mock

Page 94: Succeeding in the Biomedical Admissions Test (BMAT): A practical guide to ensure you are fully

88

101;23220

ABCDE

Question 10How many times more likely is a male to develop lung cancerthan a female?

YOU ARE NOW A QUARTER OF THE WAYTHROUGHTHIS SECTION

A 0.09B 0.9C 0.045D 0.45E 9.5

Chapter 6

Page 95: Succeeding in the Biomedical Admissions Test (BMAT): A practical guide to ensure you are fully

89

-D

-B

BPP~LEARNING MEDIA W

-

Question 12Healthcare workers are at an increased risk of both fatal andnon-fatal injuries, due to various factors ranging from violencewithin the workplace to apparatus being placed carelessly suchas syringes and needles. Violence is increasing in severity andfrequency in areas such as pharmacies, hospitals and communitycare facilities, and has now become a seemingly never-endingproblem. Ithas been proposed that a complete resolution to such

-A

Question 11Which of the following pie charts is a correct representation ofthe incidence of cancer for both males and females per 100,000people?

Full BMAT mock

Page 96: Succeeding in the Biomedical Admissions Test (BMAT): A practical guide to ensure you are fully

BPP~LEARNING MEDIA W

90

Acohort study recently published inthe Journal ofHealthy Eatingby Dr Ed Smartz et al explores the degree to which intelligenceinfluences an individual's development of initiatives to promote

Although the underlying reasons are unclear, research conductedto date indicates an individual's degree of intelligence correlateswith their degree of health and lifespan. It is thought thatintelligence can influence the development of various approachesthat can contribute to a better state of personal health andwellbeing, which include subscribing to the notion of followinga vegetarian diet.

Questions 14, 15, 16 and 17 refer to the passagebelow:Read the following passage carefully.

A James does not earn more than PeterB James earns at least as much as PeterC Peter earns at least as much as JamesD Peter earns more than James

Question 13Which two of the following statements are equivalent:

A That needles do not pose a problem to healthcare staffB Discharging mentally ill patients earlier will protect

health care workers from attackC That violence to healthcare staff isbeing caused by chronically

mentally ill patientsD Violence in hospitals is on the decreaseE Syringes and needles are the same thing

What is the main assumption in the above argument?

a problem would be to restrict the premature discharge of thechronically mentally ill from professional care services.

Chapter 6

Page 97: Succeeding in the Biomedical Admissions Test (BMAT): A practical guide to ensure you are fully

91BPP~LEARNING MEDIA W

Intervention by public health authorities to encourage thefollowing of consistently healthy lifestyles may have positiverepercussions. Such an initiative could begin with parents beingurged to encourage their children to eat more healthily, with theultimate aim of them translating their healthy eating habits intoadulthood. At the same time adults could be encouraged throughpositive education to realise the benefits of healthy eating andhow these habits can be formed during childhood.

So looking back to the original question asked by Smartz et al;does diet influence childhood intelligence and subsequent healthin adulthood? A strong relationship was found in the British 1958birth cohort, where a healthy diet score (which was based on theratio of fresh fruit versus fried food consumption at the age of33), together with a high educational achievement, significantlyaccounted for the association between childhood intelligenceand becoming a vegetarian between the ages of 18-40 years.Therefore, if diet does have an effect on intelligence, whichsubsequently results in following what can be considered a morehealthy, vegetarian lifestyle, this should surely lead to a decreasein health complications such as cancer and diabetes.

A retrospective analysis of five separate studies conducted hasshown significantly (results normalised for smoking, age andsex) that individuals who follow a vegetarian diet are 69 per centless likely to die than those that follow a non-vegetarian diet. Aseparate double blind study identified that higher ingestion offoods such as vegetables, wholemeal-based foods and certaintypes of fruit were linked to a reduced total mortality as a resultof heart disease and lower incidences of cancer in individualsover 30 years old.

their own healthy lifestyle. In this study comprising 11,000French males and females, it was shown that there was a positivecorrelation between childhood intelligence and going on to followa subsequent vegetarian diet later in life. This relationship wasshown to be independent of other potential influencing factorssuch as social class or academic achievement.

Full BMAT mock

Page 98: Succeeding in the Biomedical Admissions Test (BMAT): A practical guide to ensure you are fully

BPP~LEARNING MEDIA W

1994199119581992

ABCD92

Question 16According to the passage, in what year were the individuals inthe British 1958 cohort assessed for healthy diet scores?

A Public health authorities are powerless to make a differenceto the quality of life in adolescents through encouraginghealthy lifestyles

B Healthy eating and intelligence do not mean an individualwill follow a vegetarian lifestyle

C Bybeing vegetarian individuals are more likely to be affectedby heart disease and cancer

D The more intelligent an individual is the more likely theyare to follow a healthy lifestyle later in life

Question 15What is the main assumption made in the article above?

A Encouraging children to lead a healthy lifestyle will meanthey live longer and are less likely to suffer from braindisease

B The more intelligent an individual the less likely they willfollow a vegetarian diet in later life

C The more intelligent an individual in childhood the morelikely they are to follow a vegetarian diet resulting in reducedrisk of heart disease and cancer

D Public health authorities have no obligation to intervenein the health of children and therefore are powerless toimprove their quality of life

Question 14Which of the below options best summarises what the abovearticle infers?

Chapter 6

Page 99: Succeeding in the Biomedical Admissions Test (BMAT): A practical guide to ensure you are fully

93BPP~LEARNING MEDIA W

YOU ARE NOW HALFWAYTHROUGH THISSECTION. YOU SHOULD IDEALLYHAVE30MINUTES

REMAINING FOR THIS SECfION.

A 15.0B 15.1C 14.9D 13.9E 14.1

Question 18There are 10 players in a netball team. The oldest is 16 yearsold, and the youngest 13 years old. There are twice as many1S-year-olds as there are 14-year-olds, and three times as many16-year-olds as 13-year-olds. The median and modal age is 15.What is the mean age of the team?

A 11,000French males were analysed as part of the cohortstudy

B Results of the retrospective analysis of five studies werenormalised for smoking, age and sex

C One of the studies described was published in the Journalof Eating Healthily

D 69% of individuals are more likely to die if they follow avegetarian diet

Question 17Which of the following is the only true statement based on theinformation given in the passage?

Full BMAT mock

Page 100: Succeeding in the Biomedical Admissions Test (BMAT): A practical guide to ensure you are fully

BPP telLEARNING MEDIA W

30,00010,000Can't tell50,00060,000

ABCDE94

Question 20A bank has 100,000 customers, 60% of which have currentaccounts and 50% of which have credit cards. The bank sends aletter out to inform customers of rising interest rates. What is themaximum number of people who might receive two letters?

A 33%B 50%C 20%D 25%E 30%

What was the average increase in price of crude oil between theyears 2007 and 2008?

2008 2009200720062005Year

200420032002

~1oo!:t"0 90.. 80"2 70CJ'0 60~ 50...c'0 40-;;0CJ..."f!..~

2001

Question 19The graph below shows the average cost of a barrel of crude oil($) for a given period.

Chapter 6

Page 101: Succeeding in the Biomedical Admissions Test (BMAT): A practical guide to ensure you are fully

95BPP~LEARNING MEDIA W

A The average age of the population is reducing due tomortality

B The population dynamics have changed over a 50-yearperiod

C The percentage of elderly individuals in the population isincreasing as they are living longer

D The number of young people has decreased due to reducedfertility

What can be inferred from the above?

Question 21The population of the United Kingdom isageing. Ithas increasedfrom 55.9million in 1976to 60.2million in 2011.That is almostan 8% increase. However, this change is not spread out evenlyacross all age groups. In the last 30 or so years, the populationaged 65 or over has increased from 13% to 16%.However, thepercentage of the population under the age of 16 has declinedfrom 25%in 1976to 19%in 2011.Over the last 30years, the meanage of the UK population has increased from 34.1 years in 1976to 38.8 in 2011.This is principally due to reduced mortality inthe older generation due to healthy living.

Full BMAT mock

Page 102: Succeeding in the Biomedical Admissions Test (BMAT): A practical guide to ensure you are fully

96

Alice and KatieJanet and AliceDerek and JanetNorman and JanetPeter and Janet

ABCDE

Which two people is June sitting next to?

Katie is sitting to the right of Alice with Peter to her left.Norman is to the left of Angela.Peter is sitting opposite Derek.Derek is sitting next to Katie and Janet.Angela is sitting next to Norman and Peter and oppositeKatie.

Alice

Norman

They sit at a round table like the one below where the seatingof certain individuals is illustrated.

Question 22Eight friends go out for a meal.

Chapter 6

Page 103: Succeeding in the Biomedical Admissions Test (BMAT): A practical guide to ensure you are fully

97BPP~LEARNING MEDIA \Q

A £25,000B £33,000C £30,000D £27,000E £17,000

Question 23What is the average salary of the staff before tax?

The annual salaries are given before income tax which iscalculated at 20%.

Annual salary (£) Monthly outgoings (£)

James 42,000 2,200

Peter 25,000 1,900

Sarah 36,000 2,000

louise 21,000 1,500

Frankie 25,000 1,400

John 30,000 1,800

Kevin 31,000 1,700

Questions 23, 24, 25 and 26 refer to the dataprovided in the table below.The table below shows the salaries and monthly outgoings ofthe staff of a small company:

Full BMAT mock

Page 104: Succeeding in the Biomedical Admissions Test (BMAT): A practical guide to ensure you are fully

6

98

KevinJamesPeterJohnFrankie

ABCDE

Number of years worked

3 542

iii 15000::scc10000'"5000

00

~ 35000

~ 30000

!25000

~ 200001/1

40000

45000

Question 25The longer the staff have worked with the company the greatertheir pay. According to the graph below who is grossly overpaid?

A Louise and JohnB Peter and LouiseC James and LouiseD Jrunes and JohnE Peter and John

Question 24Whose monthly outgoings are greater than their income aftertax?

Chapter 6

Page 105: Succeeding in the Biomedical Admissions Test (BMAT): A practical guide to ensure you are fully

99BPP~LEARNING MEDIA W

Question 28To qualify for Maternity Allowance an employee must be, or haverecently been, either an employed or self-employed earner. Themajority of people who qualify for leave will also qualify for pay,and vice versa. All employees who are parents to new babieshave a right to statutory leave with pay. However, there are afew legal clauses. For instance, the privileges relating to time offfor antenatal care, to maternity leave and to protection againstdetriment or unfair dismissal in connection with maternity leavedo not apply to the following groups: members of the policeforce, MPs, the judiciary and certain types of company directors,or to masters or crew members engaged in share fishing paidsolely by how much stock they have caught.

YOU ARE NOW THREE QUARTERS OF THE WAYTHROUGH THIS SECTION

What is the final product?

126 2/9 x 31,4 6/13 400% 5/12 less 62.5% «z 1/7 -39of this of this of this of this 20% of this

Question 27Starting with the whole number in the furthest left columncalculate the correct answer by following each instruction.

A £400B £600C £425D £154E £40

Question 26Louise receives a 10% pay rise. After income tax has beendeducted together with her monthly outgoings, how much canshe expect to save each month?

Full BMAT mock

Page 106: Succeeding in the Biomedical Admissions Test (BMAT): A practical guide to ensure you are fully

100

B

A At least 6people have both a Saturday and Sunday newspaperdelivered.At least 6people do not have a Saturday or Sunday newspaperdeliveredNo more than 18 people have a magazine and Sundaynewspaper delivered

C

Which two of the statements below must be true?

Question 30A local newsagent delivers papers to 60 houses. He delivers atleast one item, but no more than three items, to each house. 50%of the houses have a Sunday paper delivered, 60% a Saturdaynewspaper, and 30% a weekly magazine delivered.

What is the connection fee and standard rate per minute?

Question 29A mobile phone company charges its customers a standardconnection fee for each call made together with a standard rateper minute. A customer made two phone calls in a day, one forsix minutes and was charged £2.65, and one for 15minutes andwas charged £5.80.

A Firemen are excluded from privileges such as maternityleave and protection against unfair dismissal

B You can only qualify for maternity leave if you have beenemployed

C The rules relating to maternity leave do not apply tomembersof the police force

D All fishermen are entitled to time off for antenatal careE All types of company directors are excluded from maternity

leave

Which one of the following statements is true?

Chapter 6

Page 107: Succeeding in the Biomedical Admissions Test (BMAT): A practical guide to ensure you are fully

101BPP~LEARNING MEDIA W

Question 32Millyrowed on an indoor rowing machine for one hour. In thefirst 20 minutes she rowed 6,OOOm.Her muscles then beganto become tired and the distance she rowed in the next 20minutes decreased by 5%.When she had 20minutes left to gothe adrenaline in her system kicked in and despite fatigue shemanaged to increase the distance travelled by 10%compared tothe distance she rowed between 20 and 40minutes.

A The Newcastle and Scottish Brewery has made a loss of19%compared to last year's net profits

B The reason targets have been reached by Newcastle andScottish Brewery is due in part to there being a RugbyWorldCup

C An upturn in the weather will ensure that Newcastle andScottishBreweryhit their target

D It is unlikely that Newcastle and ScottishBrewerywill hittheir targets this year

E Badweather in the UShas caused a loss in revenue for theNewcastle and ScottishBrewery

Whichone of the followingbest describes the main conclusionof the passage?

Question 31TheNewcastleand ScottishBreweryhasblamedtheunpredictableweather for their drop in profits for the year 2011.Statisticsshow last year's net profits of £76 million had dropped by9% in the first six months of this year compared to the sameperiod the previous year. Thewet weather has been prominentthroughout the summer and, due to the fact that there is nolarge sporting event, such as the RugbyWorldCup, to increasesales, this has led to a further 4.3%drop for the remainder ofthe year.The chairman ofNewcastleBrewerieshas claimed thatthe continuation of this bad weather in the UKand Francewillmake it most challenging to reach this year's target.

Full BMAT mock

Page 108: Succeeding in the Biomedical Admissions Test (BMAT): A practical guide to ensure you are fully

102

ocB

A The distance travelled between 0 and 10 minutes is halfthat travelled between 30 and 40 minutesThe lorry was travelling twice as quickly during 30-40minutes as during 0-10 minutesThe average speed between 30 and 40 minutes is twice thatof the average speed between 0 and 10 minutesThe lorry travelled quicker during the first 10minutes thanit did during 30-40 minutes of its journey

Which of the two following statements must be true:

The area labelled 1 is half that labelled 2.

Time (minutes)9080706050403020

2

10

80

70

60:[ 50E:;; 40.,It 30VI

20

10

00

Question 33The following graph shows the journey made by a lorry deliveringgoods to a supermarket.

A 4.2%B 5.7%c 2.7%D 4.5%E 2.9%

How much further did Milly row in the final 20minutes comparedto the first 20 minutes?

Chapter 6

Page 109: Succeeding in the Biomedical Admissions Test (BMAT): A practical guide to ensure you are fully

103BPP~LEARNING MEDIA W

A 7B 8C 5D 0E 3

What is the final digit of the code?

The sum of the digits is 29

The fourth digit plus the first digit is the same as the thirddigit

The third digit is directly below the second

The first digit is 2

No digit appears more than once in the code

The store cupboard code has 5 digits. He knows that:

1 2 3

4 5 6

7 8 9

0

Question 34A security guard has a code for the door to a store cupboard. Hehas many codes and struggles to remember them all. Instead ofwriting the codes down he writes clues to help him rememberthe codes.

Full BMAT mock

Page 110: Succeeding in the Biomedical Admissions Test (BMAT): A practical guide to ensure you are fully

BPPteILEARNING MEDIA W

104

They are all attending spinning classesIt is a SundayThey must be at the Riversmeet Leisure CentreThe time must be 9.30pmIt is a Saturday

ABCDE

On one occasion all three individuals are present in a changingroom at one of the above venues at the same time. Assumingthat each individual above must use the changing facilitieswhenever they visit any of the above facilities, which one ofthe following is true

Question 35James uses SmartAbs gym seven days a week, visits ProBowl togo bowling on aWednesday and Friday and swims at RiversmeetLeisure Centre on Sunday. Joe has a weekend only membershipat SmartAbs gym where he only ever swims. During the weekhe goes to Riversmeet Leisure Centre for spinning classes on aMonday, Wednesday and Friday and goes bowling at ProBowlevery Saturday night. John has a student membership at SmartAbsgym meaning he is restricted to using the gym Sunday toThursday between the hours of 9am and 4pm, is a member ofProBowl which he visits every Saturday to play pool withoutfail and enjoys attending spinning classes on a Wednesday atRiversmeet Leisure Centre.

Chapter 6

Page 111: Succeeding in the Biomedical Admissions Test (BMAT): A practical guide to ensure you are fully

105

Question 3Isotopes are defined as atoms which have the same number ofprotons but different numbers of neutrons.

A 38/132B 19/66C 47/144D 19/72E 5/12

Question 2Apacket of wine gums contains 3 red, 4 black and 5 green gums.Two gums are taken at random. What is the probability theywill both be the same colour?

(i) Neutralises stomach acid(ii) Converts proteins into amino acids(iii) Is called amylase(iv) Produces fatty acids

From the above enter the chemical which:

A CarbohydraseB BileC ProteaseD Lipase

Mock Section 2: Scientific Knowledgeand Applications - 30 minutesQuestion 1The following are chemicals involved in digestion:

Full BMAT mock

Page 112: Succeeding in the Biomedical Admissions Test (BMAT): A practical guide to ensure you are fully

BPP~LEARNING MEDIA W

A Muscles in the arm contract

B Impulses pass to the Sensory Neurons

C Impulse transmitted along the motor neuronDReceptors are activated

E Impulse passes through the Spinal Cord

106

Question 4A young girl places her hand in hot water. She immediatelyjumps back. Place the following series of events in the correctorder, starting with the stimulus (heat) and finishing with theresponse (to move her hand).

Mass Atomic number

A 12 5

B 7 12

C 12 7

D 13 6

E 6 13

Carbon 12 is the most common isotope of carbon. Consideringthe mass and atomic numbers, which of the options given belowcould also be an isotope of carbon?

Chapter 6

Page 113: Succeeding in the Biomedical Admissions Test (BMAT): A practical guide to ensure you are fully

107

Force(N)= Change in momentum (kgm/s) -;-Time(s)

BPP~lEARNING MEDIA W

and

where m =mass of the bulletu = initial velocity of the bulletv = final velocity of the bullet

Change in momentum = mv - mu

Thechange inmomentum canbe calculatedusing the followingequations:

Question 7Abullet is fired from a gun. An average forceof S,OOONacts for0.01seconds on a bullet with a mass of SOg.

A By controlling the amount of ions such as sodiumreabsorbed

B By releasingADHC By removing waste products from the bodyD By regulating the amount of water reabsorbed into the

bloodE Byremoving potassium from the blood

Question 6The kidneys are important organs responsible for maintainingthe homeostatic balance of body fluids. How do the kidneysmaintain blood pressure?

Rearrange the formula to make r the subject.

Question 5Thevolume of a cone is given by the formula:

Full BMAT mock

Page 114: Succeeding in the Biomedical Admissions Test (BMAT): A practical guide to ensure you are fully

BPP~LEARNING MEDIA W

108

5/6b -1/2a3/2a - 7/6b-1/6b + 1/2a4/6b - 1/2a2/3b + a

ABCDE

---; _____,~A = a, and OB = b, what in terms of a and b is the vectorxv.

Y is the midpoint between AB,X lies on the line so that OX:OB= 2:1.

O~----------------~A

B

Question 8The diagram below shows a triangle GAB.

A l,OOOm/sB 1m/sC 2,500m/sD 250m/sE 100m/s

At what speed is the bullet travelling when it is fired?

Chapter 6

Page 115: Succeeding in the Biomedical Admissions Test (BMAT): A practical guide to ensure you are fully

109BPP~LEARNING MEDIA '\'Q

A BOOkJB -BOOkJC 1,OOOkJD -l,OOOkJE SOOkJ

Bond Energy fkJ/mole)

c-c 350

C-H 400

O-H 450

0=0 SOO

c=o 800

C=C 600

Question 9Use the table below to calculate the energy change of the reactionbelow.

Full BMATmock

Page 116: Succeeding in the Biomedical Admissions Test (BMAT): A practical guide to ensure you are fully

. BPP~LEARNING MEDIA W

110

26 + 4>(10cm223 + 4>(15cm2

26 + 4n5 cm223 + 4n5 cm223 + 2...[15cm2

ABCDE

Question 11A square has a length 215 + >(3 cm. What is the area of thesquare?

o Unaffected Female

D Unaffected Male

• Affected Female

Affected Male

A 3+4+6+7B 3+4+6C 3+4+6+7+8D either 1 or 2 and either 3 or 4 + 6E either 1 or 2 and 3 + 4 + 6

Question 10Cystic fibrosis is a genetic disease which causes an excess ofmucus to be produced in the lungs. It is caused by a faulty genewhich is recessive. Study the family tree below. Which familymembers are definitely carriers of the faulty gene?

Chapter 6

Page 117: Succeeding in the Biomedical Admissions Test (BMAT): A practical guide to ensure you are fully

111BPP~LEARNING MEDIA W

A 14 ~ 5 ~ 11 ~ 4 ~ 10 ~ 1B 13 ~ 5 ~ 11 ~ 4 ~ 10 ~ 1C 15 ~ 2 ~ 7 ~ 3 ~ 6 ~ 13D 15 ~ 2 ~ 3 ~ 4 ~ 5 ~ 1E 1 ~ 10 ~ 4 ~ 11 ~ 5 ~ 14

Which route is taken by oxygenated blood returning to the heartfrom the lungs?

9

Question 13The diagram below shows a human heart.

A 1,400NB 4,900NC 500ND lOONE 980N

Question 12A car has a mass of 700kg. It accelerates from 5m/ s to 10m/ sin7 seconds. What is the driving force of the engine?

Full BMAT mock

Page 118: Succeeding in the Biomedical Admissions Test (BMAT): A practical guide to ensure you are fully

BPP~LEARNING MEOlA W

A>B>CA>B but <CB>A but <CC>A but <BA<C but >B

ABCDE112

Which two of the following statements are true about the volumesof the glasses?

.._ :;;

_..

cBA

Question 15Below are three wine glasses. All have a diameter of Bcm andthe depth of Band C is 4cm. (Take rt to equal 3.)

Slave Cylinder

-+-'---Break pad

Breakdisc

Area2m2

Hydraulic liquid

Area O.lm2

Master Cylinder

5N

Question 14The diagram below shows a hydraulic breaking system. If a forceof 5N is applied to the pedal, what force will be transferred onto the break pad?

Chapter 6

Page 119: Succeeding in the Biomedical Admissions Test (BMAT): A practical guide to ensure you are fully

113

70

BPP~LEARNING MEDIA W

A They are sensitive to temperature and pHB They are used in the cheesemaking industryC The higher the temperature, the quicker they workD They increase the activation energy of a reactionE They are used in washing powder

Question 17Enzymes are biological catalysts which occur naturally in ourbodies. Which of the following statements about enzymes aretrue:

A 100mB 200mC l,300mD BOOmE l,200m

What distance was travelled by the cyclist?

Time (seconds)605040302010

25

20

Ii'! 15

~~ 10•>

5

00

Question 16The graph below shows a journey a cyclistmade.

Full BMAT mock

Page 120: Succeeding in the Biomedical Admissions Test (BMAT): A practical guide to ensure you are fully

BPP~LEARNING MEDIA W

114

E

D

C

B

It raises blood cholesterol and blood pressure which canlead to arteriosclerosisThe nicotine the cigarettes contain stops the heart fromworking properlyCarbon monoxide released by the cigarettes reduces theamount of oxygen the red blood cells can transport totissuesIt prevents people exercising. This may lead to obesitywhich is a risk factor for CHDTar builds up in the arteries, reducing the flow of blood tothe heart

A

Why does smoking increase the risk of CHD? (Selectthe answerswhich are true.)

Number of cigarettes smoked daily25+15·251·14o

o

; 1200Es 1000o8800~~ 600Q.

~ 400

~ 2002:

CUnder45.Aged 45·55DAge55·65

1400

Question 19The table below shows the effect of smoking and age on thedeath rate from coronary heart disease (CHD).

Question 18Calculate the mass in grams of iron produced when 40g of ironIII oxide is reduced completely by aluminium. (M,Fe = 56, 0= 16, AI = 27)

Chapter 6

Page 121: Succeeding in the Biomedical Admissions Test (BMAT): A practical guide to ensure you are fully

115

Question 21What values of a, b, c and d are required to balance the followingequation?

A 35 daysB 5 daysC 7 daysD 10 daysE 9 days

What is the half life of the substance?

600

500

"D 400"0~3 300":::I8"~ 200Ua'6..a: 100

00 5 10 15 20 25 30 35 40

Time (days)

Question 20The graph below shows the decay of a radioactive substance.

Full BMAT mock

Page 122: Succeeding in the Biomedical Admissions Test (BMAT): A practical guide to ensure you are fully

116

Question 25Solve the following equation

1 _ (3x + 7) = (x + 4)2 4

A Repopulation of endangered speciesB Creating genetically modified cropsC Asexual reproductionD Generating new tissues and organs for transplantsE IVF

Question 24Molecular cloning is used in medicine to create identical copiesof a DNA sequence. Which of the following is not an applicationor form of cloning?

Question 23A step down transformer has two coils. The voltage across theprimary coil, which has 1,000turns, is 230V.Calculate the voltageacross the secondary coil which has 100 turns.

. Slood sugar Hormone Effect of hormonelevel released

A High Insulin Converts glucose into glycogen

B High Insulin Converts gtycogen into glucose

C High Glucagon Converts glycogen into glucose

D Low Glucagon Converts glycogen into glucoseo.

E Low Insulin Converts gulcose into glycogen

Question 22After eating ameal our blood sugar levels change. What happensin the body to control the blood sugar levels after eating?

Chapter 6

Page 123: Succeeding in the Biomedical Admissions Test (BMAT): A practical guide to ensure you are fully

117BPP~LEARNING MEDIA W

E Chromosomes condense andbecome visible

o Nuclear Envelope disappears

C Chromosomes align on theequator of the spindle

B Chromatids separate

A Cytoplasm divides

Question 26Theboxes labeled A-E below show the events involved in mitosis.List the events in the correct order beginning with E.

Full BMAT mock

Page 124: Succeeding in the Biomedical Admissions Test (BMAT): A practical guide to ensure you are fully

BPP~LEARNING MEDIA W

118

30.15 0.15

40.1 .;! 0.1

O.OSs s1 ;:::r ·1.5 -1 '0.5 .: - 0.5 1.5 ~ ·1.5 1.50 .':0.05

::r0-.'

·0.1 ·0.1

-0.15 .Q.15VOltllge (V) Voltage (V)

0.25 2 0.151 0.2

0.15·

0.1

s ~;: ;:~ ·1.5 -1 0.5 1.5 ~ ·1.5 1.5::r ::r0 0

·0.1

·0.15

·0.2

·0.25 ·0.15Voltage (V) Vollege (V)

Use the table to identify the components that give the solid linesin the graphs below.

Question 27The graphs below show the resistance of certain electricalcomponents. The dotted line in each graph represents a controlusing normal wire at constant temperature, and no othercomponents. Here the current is directly proportional to thevoltage.

Chapter 6

Page 125: Succeeding in the Biomedical Admissions Test (BMAT): A practical guide to ensure you are fully

119BPP~LEARNING MEDIA W

Present examples to illustrate where conditions of mindand body are directly connected,and where they must bedealt with separately.

What do you understand this statement to mean?

(c. JeffMiller)

1 'Body and soul cannot beseparatedfor purposesof treatment,for they are one and indivisible. Sick minds must be healedas well as sick bodies'.

ANSWER ONE OF THE QUESTIONS BELOWIN THESPACEPROVIDED ON THE ANSWER SHEET.

Mock Section 3: Writing Task - 30minutes

1 2 3 4

A Diode Reduced Filament Increasedtemperature bulb wire

length

B Increased Diode Filament Increasedwire bulb temperaturelength

C Decreased Filament Diode Reducedwire bulb temperaturelength

D Increased Filament Diode Decreasedtemperature bulb wire

length

E Increased Diode Filament Reducedwire bulb temperaturelength

Full BMAT mock

Page 126: Succeeding in the Biomedical Admissions Test (BMAT): A practical guide to ensure you are fully

BPP telLEARNING MEDIA W

120

.Please note in the exam you will have a choice of four .g_ue~tions.,__.

How far do you believe that knowledge and experience aredependent on one another?

Give examples of how different levels of knowledge andexperience, or lack thereof, could affect patient care.

Explain in your own words what this statement suggests.

3 'A person cannot know more than they have experienced;without experience,a person cannot understand that whichthey know',

To what extent should a clinician prioritise action overinvestigation?

Demonstrate through examples where acting on a smallamount of knowledge may be less helpful than not actingon a wealth of knowledge.

What do you think the above statement implies?

(KahlilGibran)

2 'A little knowledge that acts is worth infinitely more thanmuch knowledge that is idle',

How would you propose the conflict between the treatmentof physical and psychological illnesses be resolved?

Chapter 6

Page 127: Succeeding in the Biomedical Admissions Test (BMAT): A practical guide to ensure you are fully

• 0<...I <...I

Chapter 7Full BMATmock testanswers andjustifications

Page 128: Succeeding in the Biomedical Admissions Test (BMAT): A practical guide to ensure you are fully

BPP~LEARNING MEDIA W

122

The concluding sentence states that this form of treatment maybe a potential solution to the increasing problem of antibiotic

A specific treatment using the cream could prove amajor advancementin the battle against antibiotic resistant bacteria

B Correct

The passage suggests that due to the nature of viruses replicating,the cream may only need to be applied once and that repeatedtreatments may not be necessary which contradicts the statementabove. This option is therefore incorrect.

A major advantage of the cream described is that it will only need tobe applied twice, saving time and money

Question 2A Incorrect

Therefore, the correct answer is C

26 = 2 x 2 x 2 x 2 x 2 x 2

= 64 players

There are two players in each round, so there must be

Mock exam paper answers: Section 1Question 1Satoshi Ishii beat 6 players, therefore he must have played 6rounds

Full BMAT mock test answers andjustifications

Chapter 7

Page 129: Succeeding in the Biomedical Admissions Test (BMAT): A practical guide to ensure you are fully

123BPP~LEARNING MEDIA W

1. Younger people are more likely to apply to university:As the differencebetween the number of younger peopleapplying to university (20 and under), in 2006 and 2007and older people (21and above) is so significant, we candeduce that this statement is true.

Question 3Takingeach point in turn:

The correct answer is B.

As the passage states a bacteriophage is a form ofvirus and notbacteria this option is therefore incorrect.

The bacteriophage technology used is aform of bacteria

E Incorrect

The opening sentences state that the cream could be madeavailablein the next two years and is at ahighly advanced stageof development. This contradictswhat this option states and istherefore incorrect.

The cream is in the early stages of development

D Incorrect

Thepassageclearlystatesthat thecreamisa formofbacteriophagevirus that can be used against bacterial infections not viralinfections.This option is therefore incorrect.

The cream will have uses in treatments against viral infections

C Incorrect

resistancein bacteria.This option thereforebest describeswhatis inferred by the passage.

Full BMAT mock test answers and justifications

Page 130: Succeeding in the Biomedical Admissions Test (BMAT): A practical guide to ensure you are fully

BPP~LEARNING MEDIA \Q

124

The theme of the passage revolves around the serious threatCHD poses to industry and certainly does not paint a pictureof CHD being an issue that can be left alone. This option istherefore incorrect.

CHD is a condition. that does not need to be addressed

Question 5A Incorrect

Therefore, the correct answer is option B, 271

x2x46x410(4 x 6)x 1xOTotal = 271 rotations

Question 4First panel rotates:Second panel rotates:Third panel rotates:Fourth panel rotates:Fifth panel rotates:Sixth panel rotates:

The correct answer is A, only statement 1 can be proven.

2. More people are applying to University each year: Aswe are only given two years of data we cannot determinewhether this is true, as in 2005 there could have been moreapplications than in 2006, and in 2008there may be less than2007. In order to determine this we need data from at leastthree years to establish a trend, so this statement cannot beproven.

3. More females apply than males: Weare given no informationabout the split between male and female applications socannot say whether this is true or not, so the statementcannot be proven.

Chapter 7

Page 131: Succeeding in the Biomedical Admissions Test (BMAT): A practical guide to ensure you are fully

125BPP~LEARNING MEDIA W

This is not a plausible option because if death rates were reducedin CHD sufferers you would expect the efficiency to increasenot decrease. This option is therefore incorrect.

Reducing death rates in CHD sufferers will decrease efficiency inindustry

E Incorrect

£1,250million is hardly an insignificant amount of money whendescribing the cost of informal care of sufferers of CHD andtherefore this option is incorrect as it does not best describewhat the article infers.

The cost of informal care of CHD sufferers is insignificant

D Incorrect

The passage states that CHD cost industry £3,100 million in2011and is an increase of 25% of the previous year. Thereforeit would be incorrect to conclude that the passage implies thatnothing needs to be done about CHD.

To save costs industry does not need to do anything about CHD

C Incorrect

This option best describes what is inferred by the passage interms of the effects of CHD on industry becoming increasinglyworse. This option is therefore the best statement which describeswhat is inferred by the passage.

CHD poses a serious and increasing threat to industry

B Correct

Full BMAT mock test answers and justifications

Page 132: Succeeding in the Biomedical Admissions Test (BMAT): A practical guide to ensure you are fully

BPP~LEARNINGMEDIAW

126

So after 12 minutes the Honda will be lapped by the Ducatti.Therefore the correct answer is B, after 12 minutes.

The Honda will finish lap 1 after 4minutes, lap 2 after 8minutes,lap 3 after 12 minutes, and lap 4 after 16minutes.

So the Ducatti will finish lap 1 after 3 minutes, lap 2 after 6minutes, lap 3 after 9 minutes, lap 4 after 12 minutes, lap 5after 15 minutes and so on.

Time = Distance -;-Speed

= 10 -;-150

= 1/15 of an hour

= 4 minutes per lap.

For the Honda:

Time = Distance -;-Speed

= 10 -;-200

= 1/20 of an hour

= 3 minutes per lap.

For the Ducatti:

Question 6We need to calculate how long it takes each bike to completea lap:

Chapter 7

Page 133: Succeeding in the Biomedical Admissions Test (BMAT): A practical guide to ensure you are fully

127BPP~LEARNING MEDIA W

Question 9As prostate cancer only affects males, the population at risk ishalf of 60million, 30million. So to calculate what percentage ofthe male population was diagnosed with prostate cancer in2011,we first need to calculate how many men were diagnosed.

Therefore the correct answer is E.

In a population of 60million, there must be 1,200x 60 = 72,000people.

1,200in every 1,000,000are diagnosed

So if 120people in every 100,000people are diagnosed

Question 8From the graph we can deduce that 110females and 10 males,a total of 120people in every 100,000are diagnosed with breastcancer. We are told the population of the UK is 60 million. Asimple scale up calculation is required.

We know that emerald is below ruby and is more preciousthan pearl, but we do not have any information about whetherit is more or less precious than sapphire, therefore the correctanswer is A.

Sapphire

Ruby

Diamond

Question 7Using the information we are given we can list the stones inorder of preciousness:

Full BMAT mock test answers and justifications

Page 134: Succeeding in the Biomedical Admissions Test (BMAT): A practical guide to ensure you are fully

BPP~LEARNING MEDIA W128

Question 11We need to look at the original graph and determine whichcancers are more prominent:

This means that twice as many men are diagnosed with lungcancer than females. They are therefore twice as likely to developlung cancer than females. The correct answer is D.

Question 10We can deduce from the graph that 70 out of 100,000men arediagnosed with lung cancer compared with 35 out of 100,000females.

Therefore the correct answer is A.

(27,0007 30,000,000)x 100= (27 7 30,000) x 100= (9 7 10,000)x100= 9 7100= 0.09% of the male population was diagnosed with prostatecancer in 2011.

To express this as a fraction of the population of males:

So in a population of 30 million (males only) there are 900 x 30= 27,000males diagnosed.

The graph shows there are 90 in every 100,000people sufferingwith prostate cancer; this equates to 900 in every millionpeople.

We do this as we did for the previous question:

Chapter 7

Page 135: Succeeding in the Biomedical Admissions Test (BMAT): A practical guide to ensure you are fully

129

The passage clearly states that it is the early release of mentallyill patients from professional care services that is resulting in

Discharging mentally ill patients earlier will protect health care workersfrom attack

B Incorrect

The passage clearly states that healthcare workers are beingplaced at an increased risk of both fatal and non-fatal injuriesdue to a range of factors including leaving needles carelessly.This option is therefore incorrect.

That needles do not pose a problem to healthcare staff

Question 12A Incorrect

The correct answer is A.

We simply look for the chart which expresses this correctly.

Breast, lung, prostate, skin, bowel.

So the sections of the pie chart starting from the largest to thesmallest should be expressed in the following order:

For bowel = 10 + 5 = 15 people per 100,000

For prostate = 90 people per 100,000

For lung cancer 70 + 35 = 105 people per 100,000

For skin cancer 15 males + 25 females = 40 people per 100,000

For breast cancer 10 males + 110 females = 120 people per100,000

Full BMAT mock test answers and justifications

Page 136: Succeeding in the Biomedical Admissions Test (BMAT): A practical guide to ensure you are fully

BPP~LEARNING MEDIA W

130

Question 13A James does not earn more than Peter, so he could earn the

same as, or less than Peter. We can write this as James ~Peter

B James earns at least as much as Peter, so he could earn thesame as, or more than Peter. We can write this as James ~Peter

The focus of the passage is not whether syringes and needlesare the same thing so this is irrelevant to the argument relatingto violence. This is therefore an incorrect option.

Syringes and needles are the same thing

E Incorrect

The passage states that violence towards healthcare professionalsis increasing in severity and frequency. This option is thereforeincorrect.

Violence in hospitals is on the decrease

D Incorrect

The main assumption of the passage is that the violence tohealth care workers is being caused by chronically mentally illpatients who are being released early and that delaying theirrelease will reduce the high incidence ofviolence. This is thereforethe correct answer.

That violence to healthcare staff is being caused by chronically mentallyill patien ts

C Correct

increased violence to health care workers in various healthcaresettings. This option is therefore incorrect.

Chapter7

Page 137: Succeeding in the Biomedical Admissions Test (BMAT): A practical guide to ensure you are fully

131BPP~LEARNING MEDIA W

Although encouraging children to follow a healthy lifestyle maywell result in them living longer the passage does not infer thisand certainly does not mention any form of brain disease.

Encouraging children to lead a healthy lifestyle will mean they livelonger and are less likely to suffer from brain disease

A Incorrect

Question 14Which of the below statements best summarises what the articleinfers?

Therefore A and C are equivalent.

which is equivalent to A.

James s; Peter

We can restate the statement C by putting James first.

A James s; PeterB James ~ PeterC Peter ~ JamesD Peter> James

Sowe have the following inequalities:

C Peter earns at least as much as James, so he could earn thesame as, or more than James. We can write this as Peter ::2:

JamesD Peter earns more than James. We can write this as Peter>

James

Full BMAT mock test answers and justifications

Page 138: Succeeding in the Biomedical Admissions Test (BMAT): A practical guide to ensure you are fully

BPP telLEARNING MEDIA W

132

In the final paragraph it states that intervention by the publichealth authorities may have positive repercussions through anumber of initiatives which contradicts the above statement,making it incorrect.

Public health authorities have no obligation to intervene in the health ofchildren and therefore are powerless to improve their quality of life

D Incorrect

In the second paragraph it is clearly stated that there is a positiverelationship between childhood intelligence and following avegetarian lifestyle. In the third paragraph a double blind studyis described. When certain types of wholemeal foods, fruitsand vegetables are ingested by individuals lower incidencesof cancer and heart disease are observed. This option thereforebest summarises what the passage infers.

The more intelligent an individual in childhood the more likely theyare tofollow a vegetarian diet resulting in reduced risk of heart diseaseand cancer

C Correct

In the second paragraph it is stated that there is a positivecorrelation between childhood intelligence and following avegetarian diet, inferring that intelligent children are more likelyto follow a diet of this type.

The more intelligent an individual the less likely they will follow avegetarian diet in later life

B Incorrect

Chapter 7

Page 139: Succeeding in the Biomedical Admissions Test (BMAT): A practical guide to ensure you are fully

133BPP~LEARNING MEDIA W

In paragraph three individuals who follow a diet high inwholemeal and fruit content are less likely to develop heartdiseaseor cancerwhich suggests this option is incorrect.

By being vegetarian individuals are more likely to be affected by heartdisease and cancer

C Incorrect

Paragraph four states that the higher an individual's healthyeating score,which reflects the degree to which an individualfollows a healthy diet, the more likely they are to follow avegetarian lifestyleand be healthier, thereforecontradicting theaboveoption.

Healthy eating and intelligence do not mean an individual will followa vegetarian lifestyle

B Incorrect

The final paragraph clearly describes initiatives the healthauthorities could implement to promote healthy lifestyles inchildrenthrough the education ofparents to urge their childrento eat more healthily.This option is therefore incorrect.

Public health authorities are powerless to make a difference to thequality of life in adolescents through encouraging healthy lifestyles

A Incorrect

Question 15Which statement best describes the main assumption made inthe article above?

Full BMAT mock test answers and justifications

Page 140: Succeeding in the Biomedical Admissions Test (BMAT): A practical guide to ensure you are fully

BPP~LEARNING MEDIA W

134

Results of the retrospective analysis of five studies were normalisedfor smoking, age and sex

B Correct

In the second paragraph it clearly states that 11,000 malesand females were included in the cohort study. Therefore thisstatement cannot be true because there must have been at leastone female present in the cohort meaning that at the greatestthere could only be 10,999males present. This option is thereforeincorrect.

11,000 French males were analsjeedas part of the cohort study

A Incorrect

Questions 17Which of the following is the only true statement based on theinformation given in the passage?

The correct answer is option B as 1991 is the year that theindividuals would have been in their 33rd year.

Question 16According to the passage, in what year were the individuals inthe British 1958cohort assessed for healthy diet scores?

This is really the main thrust of the passage which relies onthe hypothesis stated in the opening paragraph that increasedchildhood intelligence leads to a healthy lifestyle later in life.

The more intelligent an individual is the more likely they are tofollowa healthy lifestyle later in life

D Correct

Chapter7

Page 141: Succeeding in the Biomedical Admissions Test (BMAT): A practical guide to ensure you are fully

135BPP~LEARNING MEDIA W

We know that the modal age is 15 (ie the middle value whenthe data is arranged in order). As we have an even number ofdata itmust lie between b and c. So either b and c are both 15,or b is 16 and c is 14.However we are told that the modal age(most occurring) age is 15 so b and c must be 15, as if they were16 and 14 then nobody on the team would be 15, and we knowthis is not true:

16 16 16 abc d e f 13

Question 18Weknow there are 10players, the oldest being 16and the youngest13.Weare also told that there are three times as many 16-year­oids as 13-year-olds, so there must be at least three 16-year-oldsand one 13-year-old. So if we arrange the ages in order:

In paragraph three it is stated that individuals who follow avegetarian diet are 69%less, not more likely, to die. This optionis therefore incorrect.

69% of individuals are more likely to die if they follow a vegetariandiet

D Incorrect

In the second paragraph the journal is clearly described as beingthe Journal ofHealthy Eating not the Journal of Eating Healthily.This option is therefore incorrect.

One of the studies described was published in the Journal of EatingHealthily

C Incorrect

In the third paragraph, which describes a retrospective analysis,it clearly states in brackets that the results were normalised forsmoking, age and sex. This is therefore the correct answer.

Full BMAT mock test answers and justifications

Page 142: Succeeding in the Biomedical Admissions Test (BMAT): A practical guide to ensure you are fully

136

Therefore the correct answer is B

Increase as a percentage = 0.50 x 100 = 50%

Increase as a fraction = 30 + 60 = 0.50

Increase in price = $90 - $60 = $30

Cost of oil in 2007 = $60 per barrel; cost in 2008 = $90

Question 19The average increase in price between 2007 and 2008.

Therefore, the correct answer is C, 14.9

= 149 + 10

= 14.9

Hence the mean age is the sum of the ages + the number ofplayers

16 16 16 15 15 15 15 14 14 13

There can't be any more 16- or 13-year-olds, so the rest of theteam must be 15 or 14years old. We are told that there are twiceas many 15-year-olds as 14-year-olds so there must be four 15-year-olds (a and d) and two 14-year-olds (e and f).

16 16 16 a 15 15 d e f 13

Chapter7

Page 143: Succeeding in the Biomedical Admissions Test (BMAT): A practical guide to ensure you are fully

137BPP~LEARNING MEDIA 'U

The final sentence in the passage dearly states that the oldergeneration are living longer due to increased healthy living,which accounts for the earlier statement describing an increasein the percentage of over-65s from 13 to 16%.Therefore this

The percentage of elderly individuals in the population is increasingas they are living longer

C Correct

Thepassagedescribesthechangeinpopulationdynamicsbetween1976to 2011,a differenceof 35years and not a 50-year period.This option is therefore incorrect.

The population dynamics have changed over a 50-year period

B Incorrect

The passage describes that the average age of the populationis increasing in main due to reduced mortality in the oldergeneration as a result of healthy living. The above statementtherefore contradicts the passage and is incorrect.

The average age of the population is reducing due to mortality

Question 21A Incorrect

Question 20This question canbe misleading.At firstyou might think, that'seasy!It's simply the 10%that overlap between credit cards andcurrent accounts. However, there is nothing to tell us that allthe 50%of credit card customers do not have current accountstoo, with the 10%overlap only having current accounts. Thiswould mean that 50,000of the customers could in theory haveboth credit and current accounts. So the correct answer is D.

Full BMAT mock test answers and justifications

Page 144: Succeeding in the Biomedical Admissions Test (BMAT): A practical guide to ensure you are fully

BPP&eILEARNING MEDIA W

138

Alice

Peter

Angela

Norman

Janet

We can therefore determine the following seating plan:

Angela is sitting next to Norman and Peter and oppositeKatie.

Derek is sitting next to Katie and Janet.

Peter is sitting opposite Derek.

Norman is to the left of Angela.

Katie is sitting to the right of Alice with Peter to her left.

Question 22According to the information provided:

Although the passage describes that the percentage of under-16shas decreased there is no mention that reduced fertility is thereason, which makes this option incorrect.

TIle number of young people has decreased due to reduced fertilittj

D Incorrect

statement best describes what is inferred in the passage out ofthe options available.

Chapter 7

Page 145: Succeeding in the Biomedical Admissions Test (BMAT): A practical guide to ensure you are fully

139BPP~lEA.'lNING MEDIA W

His monthly take home salary is £24,000 + 12 = £2,000

Leaving him with £30,000 - £6,000 = £24,000 after tax peryear.

For John: salary = £30,000, tax at 20% = £6,000

Her monthly outgoings are £1,500, so she is £100 short eachmonth.

Her monthly take home salary is £16,800 + 12 = £1,400

Leaving her with £21,000 - £4,200 = £16,800 after tax per year.

So for Louise: salary = £21,000, tax at 20% = £4,200

Question 24The options we are given include Louise, John, Peter and James.So we only need to calculate the data for these four people. Weneed to calculate how much money they earn after tax, and thensubsequently how much is left after their monthly outgoingsare taken out.

Therefore the correct answer is C.

We then divide this by 7 which equals £30,000

So the total of the salaries is: £210,000

Question 23To calculate the average we simply find the sum of the salariesand divide it by the number of people in the group.

June is therefore sitting next to Alice and Katie. Therefore thecorrect answer is A.

Full BMAT mock test answers and justifications

Page 146: Succeeding in the Biomedical Admissions Test (BMAT): A practical guide to ensure you are fully

BPP telLEARNING MEDIA W

140

Question 25We are not given any information in the table about how longthe staff have worked at the company, so we look for the pointwhich is furthest away from the line of best fit - somebody whoearns £30,000. We then use the original table to find who earnsthis amount of money. The answer is D, John. Be careful to readthe graph accurately as Kevin earns £31,000, and he is also anoption. This question tests your accuracy at graph reading.

His monthly outgoings are £2,200, so he manages to save £600each month.

His monthly take horne salary is £33,600 ..;-12 = £2,800

Leaving him with £42,000 - £8,400 = £33,600 after tax peryear.

For James: salary = £42,000, tax at 20% = £8,400

We can just check James to make sure we are correct:

So the answer is B, both Peter and Louise spend more than theyearn after tax.

His monthly outgoings are £1,900, so he is £233 short eachmonth.

His monthly take home salary is £20,000 ..;-12 = £1,667

Leaving him with £25,000 - £5,000 = £20,000 after tax peryear.

For Peter: salary = £25,000, tax at 20% = £5,000

His monthly outgoings are £1,800, so he manages to save £200each month.

Chapter 7

Page 147: Succeeding in the Biomedical Admissions Test (BMAT): A practical guide to ensure you are fully

141BPP~LEARNING MEDIA W

There is no reference made to firemen in the passage and thisoption is therefore incorrect.

Firemen are excluded from privileges such as maternity leave andprotection against unfair dismissal

Question 28A Incorrect

Therefore the correct answer is 36

Question 271. 2/9 of 126 = 282. 28 x 31,4= 913. 6/13 of 91 = 424. 400% of 42 = 1685. 5/12 of 168 = 706. less 20% of 70 = 567. 62.5% of 56 = 358. 35 x 277= 759. 75-39 = 36

The correct answer is E.

So she saves £1,540 - £1,500 = £40 a month.

Her monthly outgoings are £1,500 per month

She is taxed 20% on this, leaving her with £23,100 - £4,620 =£18,480 per year.

Her salary per month = £18,480 + 12 = £1,540 per month.

Question 26Louise's salary is £21,000. With a 10% increase it would rise to£21,000 + £2,100 = £23,100 per year.

Full BMATmock test answers and justifications

Page 148: Succeeding in the Biomedical Admissions Test (BMAT): A practical guide to ensure you are fully

142

Question 29If we let the standard rate be x and the connection fee be y, wecan write the information as two equations:

The passage states that some, but not all, company directorsare excluded from maternity leave. This option is thereforeincorrect.

All types of company directors are excluded from maternity leave

E Incorrect

The passage describes that fishermen engaged in share fishingpaid solely by how much stock they catch are not entitled totime off for antenatal care. This option is therefore incorrect.

All fishermen are entitled to time off for antenatal care

D Incorrect

The passage clearly states that rules relating to maternity leavedo not apply for various groups of workers including those whowork in the police force. This option is therefore true.

The rules relating to maternity leave do not apply to members of thepoliceforce

C Correct

The passage clearly states that you can qualify for maternityleave either as an employed or self-employed earner. This optionis therefore incorrect.

You can only qualify for maternity leave if you have been employed

B Incorrect

Chapter7

Page 149: Succeeding in the Biomedical Admissions Test (BMAT): A practical guide to ensure you are fully

143BPP~LEARNING MEDIA W

Question 30A 50%of60= 30people have a Sunday newspaper delivered,

and 60%of 60 = 36 people have a Saturday newspaperdelivered.There is an overlap of 6,so at least 6 peoplemustget both delivered.

B 60 - 36 = 24 people do not have a Saturday newspaperdelivered. They may however have a Sunday newspaperdelivered, we cannot say whether this is true or not.

So the connectionfee is SSpand standard rate per minute 35p

y = 2.65- 6x

Y = 2.65- (6 x 0.35)Y = 2.65- 2.10

Y = 0.55

We can then substitute x into Equation 1 to find the value ofy:

15x + 2.65- 6x = 5.809x = 5.80- 2.659x = 3.15x = 0.35

We can substitute y into Equation 2:

y = 2.65- 6x

If we rearrange Equation 1 to make y the subject:

6x + Y = 2.65 Equation 115x + Y = 5.80 Equation 2

Full BMAT mock test answers and justifications

Page 150: Succeeding in the Biomedical Admissions Test (BMAT): A practical guide to ensure you are fully

BPP~LEARNING MEDIA W

144

Although this statement may go some way to best describingthe passage, it is somewhat ambiguous and does not describe

An upturn in the weather will ensure that Newcastle and ScottishBrewery hit their tnrget

C Incorrect

Although the passage describes the lack of Rugby World Cupas a reason for affecting the targets it clearly states the targetshave not been reached. This option states that the targets havebeen reached due to the Rugby World Cup which contradictsthe passage. This option is therefore incorrect.

The reason targets have been reachedby Newcastle and Scottish Breweryis due in part to there being a Rugby World Cup

B Incorrect

Although the passage states that the Newcastle and ScottishBrewery net profits are down by 9% in the first six months itcannot be concluded that a loss of 19% has been made in thelast year. This option therefore incorrectly describes the mainconclusion of the passage.

The Newcastle and Scottish Bretoeruhave made a loss 0/19% comparedto last year's net profits

Question 31A Incorrect

Therefore A and C are both true.

C 30% of 60 = 18 people get a magazine delivered, 36 peopleget a Sunday newspaper delivered so no more than 18 canhave both delivered.

Chapter7

Page 151: Succeeding in the Biomedical Admissions Test (BMAT): A practical guide to ensure you are fully

145BPP~LEARNING MEDIA W

Therefore the correct answer is D, 4.5%

This expressed as a percentage = (270+ 6,000) x 100 = 27/6%=4.5%

She therefore rowed 6,270- 6,000= 270m more in the final 20minutes compared to the first 20 minutes.

Between 40 and 60minutes she rowed 10%further than she didbetween 20 and 40 minutes, 10% of 5,700 = 570, therefore sherowed 5,700+ 570 = 6,270m

Between 20 and 40minutes she rowed 5% less than 6,OOOm,5%of 6,000= 300 therefore she rowed 6,000- 300 = 5,700m

Question 32In the first 20 minutes she rowed 6,OOOm

The passage does not describe the state of the weather in the USand this statement does not best summarise the passage.

Bad weather in the US has caused a loss in revenue for the Newcastleand Scottish Brewery

E Incorrect

This statement best summarises the passage as a whole relatingto the fact that it is unlikely that the brewery will hit their targetsthis year.

It is unlikely that Newcastle and Scottish Brewery will hit theirtargets this year

D Correct

the passage as a whole and when compared to Option D is thelesser of the two options which best summarises the passage.

Full BMAT mock test answers and justifications

Page 152: Succeeding in the Biomedical Admissions Test (BMAT): A practical guide to ensure you are fully

BPP~LEARNING MEDIA W

146

The third digit is directly below the second; all possibilitiesare:

Question 34The first digit we know is 2.

The correct answers are therefore A and C.

This statement is false. The lorry was not travelling quicker inthe first 10minutes, itwas accelerating but it only reached a topspeed of 60mph, which is the speed the lorry was travelling atduring 30-40 minutes of its journey.

D The lorry travelled quicker during the first 10 minutes than itdid during 30-40 minutes of its jOU1'J1e1j

This statement is true. The average speed of the lorry during0-10 minutes was 30mph, and the average speed during 30-40minutes was 60mph.

C The average speed between 30 and 40 minutes is twice that ofthe average speed between 0 and 10 minutes

This is false, the average speed between 0-10 minutes may behalf of that between 30-40, but as the lorry's speed during thefirst 10 minutes was not constant this statement is not true.

B The lorry was travelling twice as quickly during 30-40 minutesas during 0-10 minutes

This is true; distance travelled is calculated by the area underthe graph.

Question 33A The distance travelled between 0 and 10 minutes is half that

travelled between 30 and 40 minutes

Chapter7

Page 153: Succeeding in the Biomedical Admissions Test (BMAT): A practical guide to ensure you are fully

147BPP~LEARNING MEDIA '"

2 + 5 + 8 + 6 = 21 (29 - 21 = 8). 8 already appears in the codeso is not possible

2 + 3 + 6 + 4 = 15 (29 - 15 = 14)not possible

2 + 4 + 7 + 5 = 18 (29 - 18 = 11)not possible

We know the total of the digits is 29 so we can calculate thefifth digit:

2 + 8 + 0 + (-2) not possible as negative numbers are notallowed.

2+6+9+7

2+5+8+6

2+4+7+5

2+3+6+4

2 + 1 + 4 + 2 not possible as 2 appears twice

The fourth digit plus the first digit (2) is the same as the thirddigit. (So the fourth digit = third digit minus 2)

2+8+0

2+6+9

2+5+8

2+4+7

2+3+6

2+1+4

Full BMAT mock test answers and justifications

Page 154: Succeeding in the Biomedical Admissions Test (BMAT): A practical guide to ensure you are fully

BPP~LEARNING MEDIA W

148

It is n Sunday

B Correct

According to the passage only Joe and John attend spinningclasses therefore this option is incorrect as it excludes James.

They are all attending spinning classes

A Incorrect

Using the above information

Venue James Joe John

SmartAbs Gym 7 days a Swims Sat and Gym Sun toweek Sun Thurs 9am-4pm

ProBowl Bowling Bowling Pool SaturdayWeds and Fri Monday

Riversmeet Swims Spinning Spinning classesleisure Centre Sunday classes, Mon, Weds

Weds and Fri

Question 35The key to answering the question is to establish the factsrelating to the answer options provided which relate to eitherthe venue, day / time of visiting or activity.A simple table helpsto clarify these facts

Therefore the correct answer is C, 5.

The correct code is 2 + 6 + 9 + 7 + 5

2 + 6 + 9 + 7 = 24 (29 - 24 = 5) only possible answer.

Chapter 7

Page 155: Succeeding in the Biomedical Admissions Test (BMAT): A practical guide to ensure you are fully

149BPP~LEARNING MEDIA W

Although James and Joe visit the SmartAbs on a Saturday thepassage clearlystates that Johnonly attends SmartAbson a Sundaywhich therefore rules out this option as the correct answer.

It is a Saturday

E Incorrect

Although the passage does not state the time that James and Joevisit the various facilities it does state that John has a studentmembership. This restricts his use to between the hours of 9amand 4pm which makes this option highly unlikely and thereforeincorrect.

The time must be 9.3Opm

D Incorrect

According to the passage at most there are only ever two ofthe individuals, Joe and John, present together which is on aWednesday.

They must be at the Riversmeet Leisure Centre

C Incorrect

This is the correct answer. Theymust be present at the SmartAbschanging room as James visits seven days a week, Joe swimsthere on a Sunday and John uses the gym on a Sunday.

Full BMAT mock test answers and justifications

Page 156: Succeeding in the Biomedical Admissions Test (BMAT): A practical guide to ensure you are fully

BPP~LEARNING MEDIA W

150

The important thing to remember here is that after one sweetis picked there is one less of this colour and one less in total,so the probability of picking another sweet the same colour isreduced.

We always present fractions in their lowest form, so the correctanswer is B.

= P(red x red) + P(black x black) + P(green x green)

= (3/12 x 2/11) + (4/12 x 3/11) + (5/12 x 4/11)

= 6/132 + 12/132 + 20/132

= 38/132

= 19/66

The probability of getting two the same colour

Question 2There is a total of 12 gums in the packet.

Mock exam paper answers: Section 2Question 1(i) The chemical that neutralizes stomach acid is (B)bile, which

is produced in the liver and stored in the gall bladder. It isan alkali and is also responsible for emulsifying fats.

(ii) The enzyme which breaks down proteins into amino acidsis protease (C). This is secreted in the stomach, pancreasand small intestine.

(iii) Amylase is a carbohydrase (A). It is produced in the salivaryglands and breaks down starch into sugar.

(iv) Lipase (D) catalyses the hydrolysis (breakdown) oftriglycerides (fats) into fatty acids and glycerol. Itis secretedby the pancreas and small intestine.

Chapter 7

Page 157: Succeeding in the Biomedical Admissions Test (BMAT): A practical guide to ensure you are fully

151BPP~LEARNINGMEDIA W

(multiply both sides by 3)(divide both sides by nh)(square root both sides)

Question 5V = 13 7t r2h3V = 7t r~r2 = 3V 17th

r =..f (3V17th)

The correct order of events is D, B, E, C, A

An impulse is transmitted to the muscles in the arm via amotor neuron resulting in the removal of the hand from thehot water.

Question 4Temperature receptors in the hand detect an increase intemperature, the receptors activate a signalling cascade whichis transmitted to the spinal cord via a sensory neuron (as it isa simple, involuntary reflex and does not need to go via thebrain).

The only option that fits both criteria is D.

Weare told that an isotope has the same number of protons butdiffering neutrons. Therefore the atomic number stays the same:6, but the mass increases.

Question 3The mass is the number of protons and neutrons. The atomicnumber is the number of protons.

Full BMAT mock test answers and justifications

Page 158: Succeeding in the Biomedical Admissions Test (BMAT): A practical guide to ensure you are fully

BPP~LEARNINGMEDIAW

152

Change in momentum = mv - mu

Question 7We are given the equation to calculate the momentum of thebullet:

So the correct answers are A and D.

Simply removing potassium is not how the kidneys maintainblood pressure. Yes, the kidneys do control the amount ofpotassium reabsorbed - however the statement 'removing' istoo vague.

E Incorrect

The amount of water reabsorbed affects the volume of theblood. An increase in blood volume results in an increase inblood pressure.

D Correct

The removal of waste products from the body does not directlyaffect blood pressure.

C Incorrect

The kidneys do not release ADH, this is released by the pituitarygland, and controls the levels of water the kidneys reabsorb.

B Incorrect

Both sodium and potassium are important ions in controllingblood pressure.

Question 6A Correct

Chapter 7

Page 159: Succeeding in the Biomedical Admissions Test (BMAT): A practical guide to ensure you are fully

153BPP~LEARNING MEDIA W

Therefore the correct answer is C, -1 16b + 1/2a

Question 8XY = XO + OA + AY

= -2 I3b + a + (112 b - a)

= -2/3b + a + 1/2b -1/2a= -4/6b + a + 3/6b -1/2a= -1/6b + 1/2a

Therefore the answer is A, 1,000 m/s

0.05v = 5,000 x 0.01

0.05v = 50

v = 1,000 m/s

making v the subject

Force (N) = 0.05v (kg m/s) + Time (5)

5,000 = 0.05v + 0.01

Force (N) = change in momentum (kg ml s) + Time (s)

We can substitute this into the second equation:

So the change in momentum = 0.05v

We know the mass of the bullet to be 50g = 0.05kg, and theinitial velocity of the bullet to be 0

FullBMAT mock test answers and justifications

Page 160: Succeeding in the Biomedical Admissions Test (BMAT): A practical guide to ensure you are fully

BPP~LEARNING MEDIA W

154

Therefore the correct answer is E, either 1 or 2 are carriers and3 + 4 + 6 are all carriers.

Question 10Both 3 and 4 must be carriers of the faulty gene, as their daughterinherited two faulty copies of the gene and suffers with cysticfibrosis. 6 must also be a carrier as 6' s son would definitely inheritone faulty gene from his affected mother, and as he has cysticfibrosis must have inherited another from his father (6). 6 musthave inherited his faulty gene from one of his parents - so either1 or 2 must be carriers. We do not have enough information tosay whether 5, 7 and 8 are carriers or not.

The correct answer is A.

So the overall energy change is -2,600 (the value is negative asenergy is required to break bonds) + 3,400 = 800kJ (heat is givenout so the reaction is exothermic).

So the energy released when the bonds of the reactants areformed is 3,400kJ

2H20 = 4 O-H bonds = 4 x 450 = 1,800

Products: CO2 = 2 C = 0 bonds = 2 x 800 = t600

So the energy required to break the bonds of the reactants is2,600kJ

202 = 2 0 = 0 bonds = 2 x 500 = 1,000

Reactants: CH4 = 4 C - H bonds = 4 x 400 = 1,600

Question 9To calculate the energy change, we need to calculate the energytaken to break the bonds of the reactants, and the energy requiredto form the bonds of the products.

Chapter 7

Page 161: Succeeding in the Biomedical Admissions Test (BMAT): A practical guide to ensure you are fully

lSSBPP~LEARNING MEDIA W

Therefore the correct answer is B.

4./25 + -19 + 2-115 + 2-115

= (4 x 5) + 3 + 4-115

= 23 + 4V15 cm2

Simplifying the above:

(2J5 x 2J5) + (v'3 x v'3) + (v'3 x 2J5) + (2J5 x v'3)

Expanding out the brackets:

Question 11The area of the square = (2J5 + v'3) (2J5 + v'3)

r r Rr or RR

Rr or RRr rRrRr or RR

1 or 2 must be Rr

~0Rr or RR Rr or RR

Full BMAT mock test answers and justifications

Page 162: Succeeding in the Biomedical Admissions Test (BMAT): A practical guide to ensure you are fully

BPP telLEARNING MEDIA W

156

Question 14A force of 5N is applied to an area of 0.lm2 in the mastercylinder. We need to calculate the pressure this transmits to theslave cylinder.

The correct answer is therefore A.

Question 13Oxygenated blood from the lungs enters the heart via thepulmonary vein (14). It enters into the left atrium (5). It thenpasses through the mitral value (11) into the left ventricle (4);from here it is pumped through the aortic valve (10), to the aorta(1) and then on to the rest of the body.

Therefore the answer is C.

= 700 x 5/7

=500N

F =mx a

We can then use the first equation to find the force:

= 5/7 m/s2

A 1 . Change in velocitycce eration = T'Ime

First we need to calculate the acceleration. Using the followingformula:

Question 12The equation we need here is F = m x a

Chapter 7

Page 163: Succeeding in the Biomedical Admissions Test (BMAT): A practical guide to ensure you are fully

157BPP~LEARNING MEDIA W

Volume of cone = 73 1t ?h

= ~ x 3 (42 x 4)

= 43

= 64 cm3

B is a cone

So volume of hemisphere = 256/2 = 128 cm3

Volume of sphere = 4/3 1t r3

= 4/3 x 3 x 43= 44

= 256 cm3

A is a hemisphere

Question 15We can take 1t to equal 3

Therefore a force of lOON is applied to the brake pad.

Force = pressure x area

=50 x 2

= lOON

So a pressure of 50 N / m2 is transmitted to the piston in theslave cylinder:

Pressure (N / m2) = Force (N) -;-area (m2)

= 5 + 0.1

= 50 N/m2

Full BMAT mock test answers and justifications

Page 164: Succeeding in the Biomedical Admissions Test (BMAT): A practical guide to ensure you are fully

BPP fetLEARNINGMEDIA W

158

Section 3: From 20 m/ s to 0 from 30 to 60 seconds (a triangle)

Section 2: A constant velocity of 20m/ s from 10-30 seconds(a rectangle)

Section 1: From 0 to 20m/ sin 10 seconds (a triangle)

Question 16To calculate the distance from a velocity (or speed) time graph wesimply need to calculate the area under the graph. In this question,it can be done by breaking the area into three sections.

Therefore the correct answers are Band E.

A A does have a greater volume than B, but B is not greaterthan C so this is false

C B does not have a greater volume than A so this is falseD C does have a volume greater than A but it is not less than

B so this is false.

Also A has a volume less than that of C but greater than thatof B. So E is correct.

C has a greater volume than A, and A has a greater volume thanB. So C has a greater volume than B. So B is correct.

Therefore

Volume of cylinder = 7t r2h

= 3 (42 x 4)

= 192 crrr'

C is a cylinder

Chapter 7

Page 165: Succeeding in the Biomedical Admissions Test (BMAT): A practical guide to ensure you are fully

159BPP~LEARNING MEDIA W

C They may work quicker at higher temperature, but thequestion does not state what the temperature is, so thisis not correct as temperatures above 37°C may cause theenzyme to denature, and not work so efficiently.

D Activation energy is the energy which must be overcome toget a reaction started. Enzymes lower the activation energyby either positioning the substrates so they are in an optimalposition to react, or by breaking covalent bonds.

Question 17The true answers are A, B, and E.

The correct answer is therefore D.

Therefore the total distance travelled is100 + 400 + 300m = 800m.

Section 3: Area of triangle = 1hbase x height= 1h(60 - 30) x 20

= 1h30 x 20

= 300m

Section 2:Area of rectangle = base x height= (30 - 10) x 20

= 20 x 20

=400m

Section 1:Area of triangle = 1hbase x height= 112 10 x 20

= 5 x 20

= 100m

Full BMAT mock test answers and justifications

Page 166: Succeeding in the Biomedical Admissions Test (BMAT): A practical guide to ensure you are fully

BPP~LEARNING MEDIA W

160

Carbon monoxide reduces the affinity haemoglobin has foroxygen, therefore reducing the amount of oxygen the red blood

C True

Nicotine causes constriction of the blood vessels; it does notdirectly stop the heart from working properly.

B False

It causes increased blood cholesterol and blood pressure, whichcauses fat to be deposited in the blood vessels (atheromas),resulting in the narrowing of the blood vessels which increasesthe risk of heart attacks and strokes.

A True

Question 19Smoking greatly increases the chance of developing coronaryheart disease (CHD); the main reasons for this are that:

The correct answer is therefore 28g

So % of a mole of Fe203 was used in the reaction. This wouldyield 112 x % ::: 28g of iron.

Moles reacted « 40/160 :::1.4

Ar (Atomic mass) of Fe::: 2 x 56::: 112

M, (Relative molecular mass) of Fe203 :::(56 x 2) + (16 x 3) :::160

Question 18Fe203 + 2AI -7 2Fe +Al203

Chapter 7

Page 167: Succeeding in the Biomedical Admissions Test (BMAT): A practical guide to ensure you are fully

161BPP~LEARNING MEDIA W

Therefore the answer is C, the half life of the substance is 7days.

Activity at 7 days = 250 c/ s

The activity has halved (500 to 250 c/ s) in 7 days.

The activity at 0 days = 500 c/ s

eg Time at 250 counts = 7 days,

The easiest way to calculate this it to take a value on the Y axisand find the corresponding time on the X axis.

Question 20Ahalf life is defined as the time taken for the radioactive countrate of a substance to fall by half of its starting leveL

The correct answers are therefore A and C.

Tar does not build up in the arteries; it builds up in the lungswhen cigarette smoke is inhaled.

E False

Lack of exercise is a risk factor for CHD, but this is irrespectiveof whether you smoke or not. Smoking does not make you lesslikely to exercise.

D False

cellscan transport. This reduces the amount of oxygen deliveredto the vital organs which can increase the risk of CHD.

Full BMAT mock test answers and justifications

Page 168: Succeeding in the Biomedical Admissions Test (BMAT): A practical guide to ensure you are fully

BPP~LEARNING MEDIA W

162

230V 1,000VS= 100

Substituting the values we know into the equation:

Number of turns on primaryNumber of turns on secondary

Voltage across primaryVoltage across secondary

Question 23The voltage and turns across the coils are related by the followingequation:

Therefore the correct answer is A.

Question 22Insulin is released when the blood sugar level is high; it causesexcess glucose to be converted to glycogen which is stored inthe liver. When blood sugar levels are low, glucagon is releasedwhich converts glycogen to glucose.

a = 1, b = 2, c = 2, d = 1

If we start with b, we have 2S's and 30's on the left and 4S'sand 60' s on the right. If we make b 2, and d 1, then that meansthe O's and S's balance, but we have 2 extra Na's on the right.If we make c 2, that makes the Na's balance and gives us 21'son the right which if a is 1 we have on the left, so

Question 21aI2+ bNa2S203 -4 cNaI + dNa2S406

Chapter 7

Page 169: Succeeding in the Biomedical Admissions Test (BMAT): A practical guide to ensure you are fully

163BPP~LEARNING MEDIA W

2 _ 2(3x + 7) = 2(x + 4)2 4

Step 1: Multiply both sides by 2:

Question 251 _ (3x + 7) = (x +4)

2 4

The correct answer is therefore E.

Question 24A Repopulation of endangered species - Yes, populations of

endangered species could be cloned to prevent extinctionB Creating genetically modified crops - Yes, this is currently

in process, eg growing disease-resistant cropsC Asexual reproduction - Yes, this is a form of cloning eg in

plants, as each time a plant reproduces asexually its offspringis an identical copy

D Generating new tissues and organs for transplants - Yes,neworgans can be grown from either stem cells or in anotherorganism

E NF - No, NF isInVitro Fertilisation, where an egg is fertilisedby a sperm in a laboratory environment and inserted intothe womb. The process does not involve cloning.

The voltage across the secondary coil is 23V

Vs = 23V

Vs = 230 x 1001,000

Full BMAT mock test answers and justifications

Page 170: Succeeding in the Biomedical Admissions Test (BMAT): A practical guide to ensure you are fully

BPP~LEARNING MEDIA W

164

The correct order is E, D, C, B, A

1. The chromosomes condense and become visible2. The nuclear envelope disappears3. Chromosomes move to the equator of the spindle4. The chromatids separate5. The cytoplasm divides

Question 26Mitosis is the process by which asexual reproduction occurs.

The correct answer is therefore 2.

x=2

Step 6: divide both sides by 7

14= 7x

Step 5: -4 both sides

18 = 7x + 4

Step 4: + 6x both sides

18 - 6x = x + 4

Step 3: Multiply both sides by 2:

9 -3x = (x + 4)2

2 _ 3x + 7 = (x +4)2

Step 2: Expand out bracket

Chapter 7

Page 171: Succeeding in the Biomedical Admissions Test (BMAT): A practical guide to ensure you are fully

165BPP~LEARNING MEDIA W

Resistance= voltage + current

InGraph 4, the solid line exhibits a smaller gradient than thecontrolwire. This means that it has a lower resistance than thecontrol as:

In Graph 3 the solid line represents a diode, as these allowonly a tiny flow of current at low voltages. Once the voltage isincreasedabovea certain level the diode switcheson and allowscurrent to flow.

InGraph 2, the solid line represents the presence of a filamentlamp. A cold bulb has a lower resistance and draws morecurrent. This causes it to increase in temperature which leadsto an increase in resistance.This is why the gradient of the lineincreaseswith increasing voltage.

An increasein resistancecanbe due to the presence of a resistoror due to a decreased wire thickness, increased wire length oran increase in temperature. In this case as it's the only optionwhich could be correct, it is an increase in temperature whichcauses the higher resistance, and hence the steeper line on thegraph.

We can check this by calculating the gradient (ie resistance)at a point, at a voltage of O.lV,the control wire has a currentof lA, a resistance of 0.1+ 1= O.10ohms.At the same voltagethe unknown component / wire has a current of O.5A,and aresistanceof 0.1+ 0.5= O.2ohms.

Resistance= voltage + current.

Question 27InGraph 1, the solid line has a greater gradient than the control.Thismeans that it has a higher resistancethan the control as thecurrent is able to flow less freely since:

Full BMAT mock test answers and justifications

Page 172: Succeeding in the Biomedical Admissions Test (BMAT): A practical guide to ensure you are fully

BPPtelLEARNING MEDIA W

166

Possible answerOne could say this statement refers to the way that the mindand body are inextricably linked, and should be treated assuch during most medical treatments. It could be arguedthat the majority of serious illnesses affect both the bodyand the mind, and that this should be considered when theclinician ischoosing the best course of treatment. For example,Huntington's disease affects the patient physically through

How would you propose the conflict between the treatmentof physical and psychological illnesses be resolved?

Present examples to illustrate where conditions of mindand body are directly connected, and where they must bedealt with separately.

What do you understand this statement to mean?

(c. Jeff Miller)

Mock exam paper answers: Section 31 'Body and soul cannot be separated for purposes of

treatment, for they are one and indivisible. Sick mindsmust be healed as well as sick bodies'.

The correct answer is therefore D.

A decrease in resistance can be due to a change in the metal usedin the wire, a decrease in the length of the wire, or a decreasein temperature. From the options we are given, it must be dueto a decreased wire length.

We can check this by calculating the gradient at a point. At avoJtage of O.OSY,the control has a current of 0.5A,and a resistanceof 0.05+ 0.5 = O.lohms. At the same voltage the test unknownhas a current of lA, a resistance of 0.05 + 1.0 = 0.05 ohms.

Chapter 7

Page 173: Succeeding in the Biomedical Admissions Test (BMAT): A practical guide to ensure you are fully

167

It is possible that the conflict between the treatment ofphysical and psychological illnesses could be somewhatsolved by embracing amore holistic approach. Increasingly,mainstream health care professionals are taking on boardholistic theories, such as the idea that health is not just anabsence of biological faults but more an all-encompassingwellbeing ofbody and soul. This is possibly the way forwardfor modem medicine.

In many cases, physical symptoms are related to a mentalcondition. For example, a patient suffering from severeanxiety may experience heart palpitations and chest pain.Treatment should focus on the mental issue - the anxiety- as this is the cause of the physical symptoms. Equally,the opposite can be true - physical conditions may causemental problems. An example of this would be a patientsuffering from sexual dysfunction who develops depressionas a result. In such cases, it is the clinician's job to decidewhich constitutes the cause and which constitutes theeffect, for a cornmon symptom of depression is sexualdysfunction. This is where the skill and experience of theclinician becomes paramount to a successful diagnosis andcourse of treatment. There are times when both physicaland psychological treatments are necessary. Treatment of analcoholic might require medication to reduce the symptomsof withdrawal, while additional medication or counsellingmay be needed to deal with the psychological causes andeffects of the illness. There are, however, instances whenthe mind and body must be treated separately. An examplewould be in the treatment of a cancer patient. The focushas to be on the treatment of the cancer itself, the physicalproblem rather than any mental problems which may arisein the patient due to the trauma ofhaving a life-threateningillness.

involuntary muscle movements, as well as psychologicallythrough depression, mood swings and memory loss.

Full BMAT mock test answers and justifications

Page 174: Succeeding in the Biomedical Admissions Test (BMAT): A practical guide to ensure you are fully

BPP~LEARNING MEDIA W

168

Any trauma doctor or paramedic needs to have the abilityto react quickly to a situation while under pressure andusually with relatively little information to work with. Theskill of a paramedic is to make the correct decision undersuch circumstances, but he or she would be calling on pastknowledge and experience, combined with their naturalinstincts. Apasser-by at the same accident scene may possessthat same instinct to help others - the difference would bethat the average passer-by would not be equipped withthe same medical knowledge. Having watched a medicaldrama or documentary the night before, the passer-by maydetermine that the victim is suffocating and in need of a

Possible answerItcould be argued that instinct is our greatest natural defenceagainst potential threats. Acting quickly, with little or noreal knowledge of a situation, could potentially mean thedifference between life and death; conversely, refusal toact for any reason - regardless of whether the individualinvolved is capable of managing the situation - could resultin unnecessary delay and even permanent damage or death.Knowing when to react instantly and when to assess thesituation before making a decision are essential skills forany medical practitioner.

To what extent should a clinician prioritise action overinvestigation?

Demonstrate through examples where acting on a smallamount of knowledge may be less helpful than not actingon a wealth of knowledge.

What do you think the above statement implies?

(Kahlil Gibran)

2 'A little knowledge that acts is worth infinitely more thanmuch knowledge that is idle'.

Chapter 7

Page 175: Succeeding in the Biomedical Admissions Test (BMAT): A practical guide to ensure you are fully

169BPP~LEARNING MEDIA W

Possible answerConfusing knowledge and experience is potentially a verydangerous thing for any medical professional to do, andthough the two concepts are inextricably linked, they cannot

How far do you believe that knowledge and experience aredependent on one another?

Give examples of how different levels of knowledge andexperience, or lack thereof, could affect patient care.

Explain in your own words what this statement suggests

3 IA person cannot know more than they have experienced;without experience, a person cannot understand that whichthey know.'

A clinician should always investigate as far as possible forany patient, but a lengthy diagnosis is not always possibleand fast action will inevitably be required. The true dangerlies in those acting on perceived knowledge or in a situationwell beyond their capabilities.Actingon the smallest amountof knowledge with no real comprehension of how it mayrelate to differing circumstances can only be dangerous,and unless the patient is certainly facing death either way,waiting for the necessary help is always the best course ofaction.

tracheotomy; realising that time is short, the passer-by maydecide that, instead of calling for appropriate help andwaiting for a medical professional, they need to act on thespot and perform the procedure themselves. They may belucky; more likely, however, is that they could puncturea vein or artery, perforate the vocal cords, or pierce theoesophagus. Where the victim may have survived longenough to receive help, they would now bleed or choke todeath before professional help could arrive.

Full BMAT mock test answers and justifications

Page 176: Succeeding in the Biomedical Admissions Test (BMAT): A practical guide to ensure you are fully

BPP~LEARNING MEDIA W

170

Based on this argument, it is certainly safe to assume thatknowledge and experience are symbiotic. It seems sensibleto suggest that experience can improve knowledge throughenabling a hands-on learning approach; though coreknowledge is ultimately vital, it is only through experiencethat we can learn how important that knowledge is. Asa doctor, experience is vital; within real-world situationsknowledge cannot be relied upon in isolation, and mustbe tempered by experience to ensure safe and effectivepractice.

Within a clinical setting, lack of knowledge or experiencecould lead to failures in patient care. A medical student,for example, must be expected to possess a certain level ofknowledge, but GUillotyet calIon experience; they must relyon the experience of more senior doctors to ensure that theirknowledge is applied safely and correctly.A trainee surgeon,regardless of how much they know, could not be expectedto perform complex heart surgery with no experience. Evenif they knew exactly what to do, lack of experience wouldrender them unable to handle unexpected complications; thus,lack of experience leads to a lack of adaptability, an essentialskill for doctors under pressured circumstances. Performingsuch an operation without any real understanding or insightwould ultimately endanger a patient and potentially lead todeath. While knowledge is finite and potentially has limits,experience is both abstract and unlimited; knowledge canbe lost or forgotten much more easily than experience.Experience can also be sought through failure, particularlywhere knowledge may be incomplete.

be simply interchanged. Knowledge could be defined asthe possession of information which ultimately leads tothe ability to perform an action, while experience could bedefined as practice in performing those actions; knowledge,therefore, tells us what to do, while experience tells us how,and provides a full understanding of the implications andapplications of the knowledge we possess.

Chapter 7